*NURSING > MED-SURG EXAM > NURSING 1040 med surg (GRADED A) bank maternity, Latest Questions and Answers solution (All)

NURSING 1040 med surg (GRADED A) bank maternity, Latest Questions and Answers solution

Document Content and Description Below

NURSING 1040 med surg (GRADED A) bank maternity A nurse in a health clinic is reviewing contraceptive use with a group of adolescent clients. Which of the following statements by an adolescent refle... cts an understanding of the teaching? A. "A water-soluble lubricant should be used with condoms." B. "A diaphragm should be removed 2 hours after intercourse." C. "Oral contraceptives can worsen a case of acne." D. "A contraceptive patch is replaced once a month." A nurse is instructing a client who is taking an oral contraceptive about danger signs to report to her provider. The nurse determines the client understands the teaching when the client states the need to report which of the following? A. Reduced menstrual flow. B. Breast tenderness. C. Shortness of breath. D. Headaches. A nurse in an obstetrical clinic is teaching a client about using an IUD for contraception. Which of the following statements by the client indicates an understanding of the teaching? A. "An IUD should be replaced annually during a pelvic exam." B. "I cannot get an IUD until after I've had a child." C. "I should expect intermittent abdominal pain while the IUD is in place." D. "A change in the string length of my IUD is expected." A nurse is teaching a client about potential adverse effects of implantable progestins. Which of the following adverse effects should the nurse include? (SATA) A. Tinnitus. B. Irregular vaginal bleeding. C. Weight gain. D. Breast changes. E. Gingival hyperplasia. A nurse in a clinic is teaching a client about her new prescription for medroxyprogesterone. Which of the following information should the nurse include in the teaching? (SATA) A. "Weight loss can occur." B. "You are protected against STIs." C. "You should increase your intake of calcium." D. "You should avoid taking antibiotics." E. "Irregular vaginal spotting can occur." A nurse in a clinic is caring for a group of female clients who are being evaluated for infertility. Which of the following clients should the nurse anticipate the provider will refer to a genetic counselor? A. A client whose sister has alopecia. B. A client whose partner has von Willebrand disease. C. A client who has an allergy to sulfa. D. A client who had rubella 3 months ago. C. "The man is the easiest to assess, and the provider will usually begin there." A nurse is caring for a couple who is being evaluated for infertility. Which of the following statements by the nurse indicates understanding of the infertility assessment process? A. "You will need to see a genetic counselor as part of the assessment." B. "It is usually the woman who is having trouble, so the man doesn't have to be involved." C. "The man is the easiest to assess, and the provider will usually begin there." D. "Think about adopting first because there are many babies that need good homes." A. Occupation. B. Menstrual history. C. Childhood infectious diseases. A nurse in an infertility clinic is providing care to a couple who has been unable to conceive for 18 months. Which of the following data should be included in the assessment? (SATA) A. Occupation. B. Menstrual history. C. Childhood infectious diseases. D. History of falls. E. Recent blood transfusions. A. "It is good to know that I won't have a tubal pregnancy in the future." A nurse in a clinic is caring for a client who is to be seen by the provider for a postoperative appointment following a salpingectomy due to an ectopic pregnancy. Which of the following statements by the client requires clarification? A. "It is good to know that I won't have a tubal pregnancy in the future." B. "The doctor said that this surgery can affect my ability to get pregnant again." C. "I understand that one of my fallopian tubes had to be removed." D. "Ovulation can still occur because my ovaries were not affected." A. January 8. A nurse is caring for a client who is pregnant and states that her last menstrual period was April 1st. Which of the following is the client's estimated date of delivery? A. January 8. B. January 15. C. February 8. D. February 15. A. Client has delivered one newborn at term. D. Client has had two prior pregnancies. E. Client has one living child. A nurse in a prenatal clinic is caring for a client who is in the first trimester of pregnancy. The client's health record includes this data: G3 T1 P0 A1 L1. How should the nurse interpret this information? (SATA) A. Client has delivered one newborn at term. B Client has experienced no preterm labor. C. Client has been through active labor. D. Client has had two prior pregnancies. E. Client has one living child. B. Goodell's sign. C. Ballottement. D. Chadwick's sign. A nurse is reviewing the health record of a client who is pregnant. The provider indicated the client exhibits probable signs of pregnancy. Which of the following findings should the nurse expect? (SATA) A. Montogomery's glands. B. Goodell's sign. C. Ballottement. D. Chadwick's sign. E. Quickening. C. "This is due to the weight of the uterus on the vena cava." A nurse in a prenatal clinic is caring for a client who is pregnant and experiencing episodes of maternal hypotension. The client asks the nurse what causes these episodes. Which of the following responses should the nurse make? A. "This is due to an increase in blood volume." B. "This is due to pressure from the uterus on the diaphragm." C. "This is due to the weight of the uterus on the vena cava." D. "This is due to increased cardiac output." D. "You should collect urine from the first morning void." A nurse in a clinic receives a phone call from a client who believes she is pregnant and would like to be tested in the clinic to confirm her pregnancy. Which of the following information should the nurse provide to the client? A. "You should wait until 4 weeks after conception to be tested." B. "You should deb off any medications for 24 hours prior to the test." C. "You should be NPO for at least 8 hours prior to the test." D. "You should collect urine from the first morning void." C. Perform the pelvic rock exercise every day. D. Use proper body mechanics. A nurse is teaching a group of women who are pregnant about measures to relieve backache during pregnancy. Which of the following measures should the nurse include in the teaching? (SATA) A. Avoid any lifting. B. Perform kegel exercises twice a day. C. Perform the pelvic rock exercise every day. D. Use proper body mechanics. E. Avoid constrictive clothing. A. Vaginal bleeding. A nurse is caring for a client who is pregnant and reviewing signs of complications the client should promptly report to the provider. Which of the following complications should the nurse include in the teaching? A. Vaginal bleeding. B. Swelling of the ankles. C. Heartburn after eating. D. Lightheadedness when lying on back. A. Eat crackers or plain toast before getting out of bed. A client who is at 7 weeks of gestation is experiencing nausea and vomiting in the morning. Which of the following information should the nurse include in the teaching? A. Eat crackers or plain toast before getting out of bed. B. Awaken during the night to eat a snack. C. Skip breakfast and eat launch after nausea has subsided. D. Eat a large evening meal. A. Breast tenderness. B. Urinary frequency. C. Epistaxis. A nurse is teaching a client who is at 6 weeks of gestation about common discomforts of pregnancy. Which of the following findings should the nurse include in the teaching? (SATA) A. Breast tenderness. B. Urinary frequency. C. Epistaxis. D. Dysuria. E. Epigastric pain. B. "It is normal to have these feelings during the first few months of pregnancy." A client who is at 8 weeks of gestation tells the nurse that she isn't sure she is happy about being pregnant. Which of the following responses should the nurse make? A. "I will inform the provider that you are having these feelings." B. "It is normal to have these feelings during the first few months of pregnancy." C. "You should be happy that you are going to bring new life into the world." D. "I am going to make an appointment with the counselor for you to discuss these thoughts." A. Dark green leafy vegetables. A nurse in a prenatal clinic is providing education to a client who is in the 8th week of gestation. The client states that she does not like milk. Which of the following foods should the nurse recommend as a good source of calcium? A. Dark green leafy vegetables. B. Deep red or orange vegetables. C. White breads and rice. D. Meat, poultry, and fish. B. 3.6kg (8lb) weight gain and is in her first trimester. A nurse in a prenatal clinic is caring for four clients. Which of the following clients' weight fain should the nurse report to the provider? A. 1.8kg (4 lb) weight gain and is in her first trimester. B. 3.6kg (8 lb) weight gain and is in her first trimester. C. 6.8kg (15 lb) weight gain and is in her second trimester. D. 11.3kg (25 lb) weight gain and is in her third trimester. D. Neural tube defects. A nurse in a clinic is teaching a client of childbearing age about recommended folic acid supplements. Which of the following defects can occur in the fetus or neonate as a result of folic acid deficiency? A. Iron deficiency anemia. B. Poor bone formation. C. Macrosomic fetus. D. Neural tube defects. D. Orange juice. A nurse is reviewing a new prescription for iron supplements with a client who is in the 8th week gestation and has iron deficiency anemia. Which of the following beverages should the nurse instruct the client to take the iron supplements with? A. Ice water. B. Low-fat or whole milk. C. Tea or coffee. D. Orange juice. D. "I will continue my calcium supplements because I don't like milk." A nurse is reviewing postpartum nutrition needs with a group of new mothers who are breastfeeding their newborns. Which of the following statements by a member of the group indicates an understanding of the teaching? A. "I am glad I can have my morning coffee." B. "I should tai folic acid to increase my milk supply." C. "I will continue adding 330 calories per day to my diet." D. "I will continue my calcium supplements because I don't like milk." B. Fetal breathing movement. C. Fetal tone. E. Amniotic fluid volume. A nurse is reviewing findings of a client's biophysical profile (BPP). The nurse should expect which of the following variables to be included in this test? (SATA) A. Fetal weight. B. Fetal breathing movement. C. Fetal tone. D. Fetal position. E. Amniotic fluid volume. A. Alpha-fetoprotein (AFP). A nurse is caring for a client who is in preterm labor and is scheduled to undergo an amniocentesis. The nurse should evaluate which of the following tests to assess fetal lung maturity? A. Alpha-fetoprotein (AFP). B. Lecithin/sphingomyelin (L/S) ratio. C. Kleihauer-betke test. D. Indirect coombs' test. D. "It awakens a sleeping fetus." A nurse is caring for a client who is pregnant and undergoing a non stress test. The client asks why the nurse is using an acoustic vibration device. Which of the following responses should the nurse make? A. "It is used to stimulate uterine contractions." B. "It will decrease the incidence of uterine contractions." C. "It lulls the fetus to sleep." D. "It awakens a sleeping fetus." C. "You should empty your bladder prior to the procedure." A nurse is teaching a client who is pregnant about the amniocentesis procedure. Which of the following statements should the nurse include in the teaching? A. "You will lay on your right side during the procedure." B. "You should not eat anything for 24 hours prior to the procedure." C. "You should empty your bladder prior to the procedure." D. "The test is done to determine gestational age." A. Decreased fetal movement. B. Intrauterine growth restriction (IUGR). C. Postmaturity. A nurse is caring for a client who is pregnant and is to undergo a contraction stress test (CST). Which of the following findings are indications of this procedure? (SATA) A. Decreased fetal movement. B. Intrauterine growth restriction (IUGR). C. Postmaturity. D. Placenta previa. E. Amniotic fluid emboli. B. Ectopic pregnancy. A nurse in the emergency department is caring for a client who reports abrupt, sharp, right-sided lower quadrant abdominal pain and bright red vaginal bleeding. The client states she missed one menstrual cycle and cannot be pregnant because she has an intrauterine device. The nurse should suspect which of the following? A. Missed abortion. B. Ectopic pregnancy. C. Severe preeclampsia. D. Hydatidiform mole. B. Blunt abdominal trauma. C. Cocaine use. E. Cigarette smoking. A nurse is providing care for a client who is diagnosed with a marginal abruptio placentae. The nurse is aware that which of the following findings are risk factors for developing the condition? (SATA) A. Fetal position. B. Blunt abdominal trauma. C. Cocaine use. D. Maternal age. E. Cigarette smoking. A. Betamethasone. A nurse is providing care for a client who is at 32 weeks of gestation and who has a placenta previa. The nurse notes that the client is actively bleeding. Which of the following types of medications should the nurse anticipate the provider will prescribe? A. Betamethasone. B. Indomethacin. C. Nifedipine. D. Methylergonovine. C. Hydatidiform mole. A nurse at an antepartum clinic is caring for a client who is at 4 months of gestation. The client reports continued nausea and vomiting and scant, prune-colored discharge. She has experienced no weight loss and has a fundal height large than expected. Which of the following complications should the nurse suspect? A. Hyperemesis gravidarum. B. Threatened abortion. C. Hydatidiform mole. D. Preterm labor. D. Report of severe shoulder pain. A nurse is caring for a client who has a diagnosis of ruptured ectopic pregnancy. Which of the following findings is seen with this condition? A. No alteration in meses. B. Transvaginal ultrasound indicating a fetus in the uterus. C. Serum progesterone greater than the expected reference range. D. Report of severe shoulder pain. A. Episiotomy. C. Forceps. E. Internal fetal monitoring. A nurse is admitting a client who is in labor and has HIV. Which of the following interventions should the nurse identify as contraindicated for this client? (SATA) A. Episiotomy. B. Oxytocin infusion. C. Forceps. D. Cesarean birth. E. Internal fetal monitoring. A. Joint pain. B. Malaise. C. Rash. E. Tender lymph nodes. A nurse in an antepartum clinic is assessing a client who has a TORCH infection. Which of the following findings should the nurse expect? (SATA) A. Joint pain. B. Malaise. C. Rash. D. Urinary frequency. E. Tender lymph nodes. A. Ceftiaxone. A nurse is caring for a client who has gonorrhea. Which of the following medications should the nurse anticipate the provider will prescribe? A. Ceftiaxone. B. Fluconazole. C. Metronidazole. D. Zidovudine. A. Gonorrhea. B. Chlamydia. C. HIV. D. Group B streptococcus beta-hemolytic. A nurse is caring for a client who is in labor. The nurse should identify that which of the following infections can be treated during labor or immediately following birth? (SATA) A. Gonorrhea. B. Chlamydia. C. HIV. D. Group B streptococcus beta-hemolytic. E. TORCH infection. D. "A woman should avoid consuming undercooked meat while pregnant." A nurse manager is reviewing ways to prevent a TORCH infection during pregnancy with a group of newly licensed nurses. Which of the following statements by a nurse indicates understanding of the teaching? A. "Obtain an immunization against rubella early in pregnancy." B. "Seek prophylactic treatment if cyomegalovirus is detected during pregnancy." C. "A women should avoid crowded places during pregnancy." D. "A woman should avoid consuming undercooked meat while pregnant." A. Obesity. B. Multifetal pregnancy. D. Migraine headache. A nurse is caring for a client who is at 14 weeks gestation and has hyperemesis gravidarum. The nurse should identify that which of the following are risk factors for the client? (SATA) A. Obesity. B. Multifetal pregnancy. C. Maternal age greater than 40. D. Migraine headache. E. Oligohydramnios. B. Urine ketones present. A nurse is caring for a client who has suspected hyperemesis gravidarum and is reviewing the client's laboratory reports. Which of the following findings is a manifestation of this condition? A. Hgb 12.2g/dL. B. Urine ketones present. C. Alanin aminotransferase 20 IU/L. D. Serum glucose 114 mg/dL. A. Respirations less than 12/min. B. Urinary output less than 30 mL/hr. D. Decreased level of consciousness. A nurse is administering magnisium sulfate IV to a client who has severe preeclampsia for seizure prophylaxis. Which of the following indicates magnesium sulfate toxicity? (SATA) A. Respirations less than 12/min. B. Urinary output less than 30 mL/hr. C. Hyperreflexic deep-tendon reflexes. D. Decreased level of consciousness. E. Flushing and sweating. D. Calcium gluconate. A nurse is caring for a client who is receiving IV magnesium sulfate. Which of the following medications should the nurse anticipate administering if magnesium sulfate toxicity is suspected? A. Nifedipine. B. Pyridoxine. C. Ferrous sulfate. D. Calcium gluconate. C. "I plan to drink more orange juice while taking this pill." A nurse is reviewing a new prescription for ferrous sulfate with a client who is at 12 weeks of gestation. Which of the following statements by the client indicates understanding of the teaching? A. "I will take this pill with my breakfast." B. "I will take this medication with a glass of milk." C. "I plan to drink more orange juice while taking this pill." D. "I plan to add more calcium-rich foods to my diet while taking this medication." A. Urinary tract infection. B. Multifetal pregnancy. D. Diabetes mellitus. E. Uterine abnormalities. A nurse is caring for a client who reports indications of preterm labor. Which of the following findings are risk factors of this condition? (SATA) A. Urinary tract infection. B. Multifetal pregnancy. C. Oliogohydramnios. D. Diabetes mellitus. E. Uterine abnormalities. D. Betamethasone. A nurse in labor and delivery is providing care for a client who is in preterm labor at 32 weeks of gestation. Which of the following medications should the nurse anticipate the provider will prescribe to hasten fetal lung maturity? A. Calcium gluconate. B. Indomethacin. C. Nifedipine. D. Betamethasone. B. Dizziness. A nurse is caring for a client who is receiving nifedipine for prevention of preterm labor. The nurse should monitor the client for which of the following manifestations? A. Blood-tinged sputum. B. Dizziness. C. Pallor. D. Somnolence. A. Fetal distress. C. Vaginal bleeding. D. Cervical dilation greater than 6 cm. A nurse is caring for a client who has a prescription for magnesium sulfate. The nurse should recognize that which of the following are contraindications for use of this medication? (SATA) A. Fetal distress. B. Preterm labor. C. Vaginal bleeding. D. Cervical dilation greater than 6 cm. E. Severe gestational hypertension. D. Keep a daily record of fetal kick counts. A nurse is reviewing discharge teaching with a client who has premature rupture of membranes at 26 weeks of gestation. Which of the following instructions should the nurse include in the teaching? A. Use a condom with sexual intercourse. B. Avoid bubble bath solution when taking a tub bath. C. Wipe from the back to the front when performing perineal hygiene. D. Keep a daily record of fetal kick counts. D. True contractions. A nurse in the labor and delivery unit receives a phone call from a client who reports that her contractions started about 2 hr ago, did not go away when she had two glasses of water and rested, and became stronger since she started walking. Her contractions occur every 10 min and last about 30 seconds. She hasn't had any fluid leak from her vagina. However, she saw some blood when she wiped after voiding. Based on this report, which of the following clinical findings should the nurse recognize that the client is experiencing? A. Braxton hicks contractions. B. Rupture of membranes. C. Fetal descent. D. True contractions. A. First stage, latent phase. A nurse in the labor and delivery unit is caring for a client in labor and applies an external fetal monitor and tocotransducer. The FHR is around 140/min. Contractions are occurring every 8 min and 30-40 seconds in duration. The nurse performs a vaginal exam and finds the cervix 2 cm dilated, 50% effaced, and the fetus is at a -2 station. Which of the following stages and phases of labor is this client experiencing? A. First stage, latent phase. B. First stage, active phase. C. First stage, transition phase. D. Second stage of labor. B. Monitor FHR for distress. A client experiences a large gush of fluid from her vagina while walking in the hallway of the birthing unit. Which of the following actions should the nurse take first? A. Check the amniotic fluid for meconium. B. Monitor FHR for distress. C. Dry the client and make her comfortable. D. Monitor uterine contractions. B. Infection. A nurse in labor and delivery unit is completing an admission assessment for a client who is at 39 weeks of gestation. The client reports that she has been leaking fluid from her vagina for 2 days. Which of the following conditions is the client at risk for developing? A. Cord prolapse. B. Infection. C. Postpartum hemorrhage. D. Hydramnios. C. Transition phase. A nurse is caring for a client who is in active labor and becomes nauseous and vomits. The client is very irritable and feels the urge to have a bowel movement. She states, "I've had enough. I can't do this anymore. I want to go home right now." Which of the following stages of labor is the client experiencing? A. Second stage. B. Fourth stage. C. Transition phase. D. Latent phase. A. Encourage use of patterned breathing techniques. C. Administer opioid analgesic medication. D. Suggest application of cold. A nurse is caring for a client who is at 40 weeks of gestation and experiencing contractions every 3 to 5 min and becoming stronger. A vaginal exam reveals that the client's cervix is 3 cm dilated, 80% effaced, and -1 station. The client asks for pain medication. Which of the following actions should the nurse take? (SATA) A. Encourage use of patterned breathing techniques. B. Insert an indwelling urinary catheter. C. Administer opioid analgesic medication. D. Suggest application of cold. E. Provides ice chips. B. Sacral counterpressure. A nurse is caring for a client who is in active labor. The client reports lower-back pain. The nurse suspects that this pain is related to a persistent occiput posterior fetal position. Which of the following non pharmacological nursing interventions should the nurse recommend to the client? A. Abdominal effleurage. B. Sacral counterpressure. C. Showering if not contraindicated. D. Back rub and massage. C. "It is needed to counteract hypotension." A nurse is caring for a client following the administration of an epidural block and is preparing to administer an IV fluid bolus. The client's partner asks about the purpose of IV fluids. Which of the following is an appropriate response for the nurse to make? A. "It is needed to promote increased urine output." B. "It is needed to counteract respiratory depression." C. "It is needed to counteract hypotension." D. "It is needed to prevent oligohydramnios." A. Pudendal. A nurse is caring for a client who is in the second stage of labor. The client's labor has been progressing, and she is expected to deliver vaginally in 20 min. The provider is preparing to administer lidocaine for pain relief and perform an episiotomy. The nurse should know that which of the following types of regional anesthetic block is to be administered? A. Pudendal. B. Epidural. C. Spinal. D. Paracervical. D. Place an oxygen mask over the client's nose and mouth. A nurse is caring for a client who is using patterned breathing during labor. The client reports numbness and tingling of the fingers. Which of the following actions should the nurse take? A. Administer oxygen via nasal cannula at 2 L/min. B. Apply a warm blanket. C. Assist the client to a side-lying position. D. Place an oxygen mask over the client's nose and mouth. A. Moderate variability. B. FHR accelerations. D. Normal baseline FHR. A nurse is providing care for a client who is in active labor. Her cervix is dilated to 5 cm, and her membranes are intact. Based on the use of external electronic fetal monitoring the nurse notes a FHR of 115 to 125/min with occasional increases up to 150 to 155/min that last for 25 seconds, and have beat-to-beat variability of 30/min. There is no slowing of FHR from the baseline. The nurse should recognize that this client is exhibiting signs of which of the following? (SATA) A. Moderate variability. B. FHR accelerations. C. FHR decelerations. D. Normal baseline FHR. E. Fetal tachycardia. B. "It can detect abnormal fetal heart tones early." D. "It allows for accurate readings with maternal movement." E. "It can measure uterine contraction intensity." A nurse is teaching a client about the benefits of internal fetal heart monitoring. Which of the following statements should the nurse include in the teaching? (SATA) A. "It is considered a noninvasive procedure." B. "It can detect abnormal fetal heart tones early." C. "It can determine the amount of amniotic fluid you have." D. "It allows for accurate readings with maternal movement." E. "It can measure uterine contraction intensity." D. Relaxation between uterine contractions. A nurse is reviewing the electronic monitor tracing for a client who is in active labor. The nurse should know that a fetus receives more oxygen when which of the following appears on the tracing? A. Peak of the uterine contraction. B. Moderate variability. C. FHR acceleration. D. Relaxation between uterine contractions. A. Assist the client into the left-lateral position. A nurse is caring for a client who is in labor and observes late decelerations on the electronic fetal monitor. Which of the following is the first action the nurse should take? A. Assist the client into the left-lateral position. B. Apply a fetal scalp electrode. C. Insert an IV catheter. D. Perform a vaginal exam. B. Palpate the fundus of the uterus. A nurse is performing Leopold maneuvers on a client who is in labor. Which of the following techniques should the nurse use to identify the fetal lie? A. Apply palms of both hands to sides of uterus. B. Palpate the fundus of the uterus. C. Grasp lower uterine segment between thumb and fingers. D. Stand facing client's feet with fingertips outlining cephalic prominence. C. "The vaginal area will bulge as the baby's head appears." A nurse is caring for a client and her partner during their second stage of labor. The client's partner asks the nurse to explain how he will know when crowning occurs. Which of the following responses should the nurse make? A. "The placenta will protrude from the vagina." B. "Your partner will report a decrease in the intensity of contractions." C. "The vaginal area will bulge as the baby's head appears." D. "Your partner will report less rectal pressure." B. Prepare for an impending delivery. A nurse is caring for a client who is in the transition phase of labor and reports that she needs to have a bowel movement with the peak of contractions. Which of the following actions should the nurse make? A. Assist the client to the bathroom. B. Prepare for an impending delivery. C. Prepare to remove a fecal impaction. D. Encourage the client to take deep, cleansing breaths. A. Lengthening of the umbilical cord. D. Appearance of dark blood from the vagina. E. Fundus firm upon palpation. A nurse is caring for a client in the third stage of labor. Which of the following findings indicate the placental separation? (SATA) A. Lengthening of the umbilical cord. B. Swift gush of clear amniotic fluid. C. Softening of the lower uterine segment. D. Appearance of dark blood from the vagina. E. Fundus firm upon palpation. D. Defer vaginal examinations. A nurse in labor and delivery is planning care for a newly admitted client who reports she is in labor and has been having vaginal bleeding for 2 weeks. Which of the following should the nurse include in the plan of care? A. Inspect the introitus for a prolapsed cord. B. Perform a test to identify the ferning pattern. C. Monitor station of the presenting part. D. Defer vaginal examinations. D. "A distended bladder reduces pelvic space needed for birth." A nurse is caring for a client who is in the first stage of labor and is encouraging the client to void every 2 hr. Which of the following statements should the nurse make? A. "A full bladder increases the risk for fetal trauma." B. "A full bladder increases the risk for bladder infections." C. "A distended bladder will be traumatized by frequent pelvic exams." D. "A distended bladder reduces pelvic space needed for birth." A. Oligohydramnios. C. Fetal cord compression. A nurse is caring for a client who is at 42 weeks of gestation and is admitted to the labor and delivery unit. During an ultrasound, it is noted that the fetus is large for gestational age. The nurse reviews the prescription from the provider to begin an amnioinfusion. Which of the following conditions should the nurse plan to prepare an amnioinfusion? (SATA) A. Oligohydramnios. B. Hydramnios. C. Fetal cord compression. D. Hydration. E. Fetal immaturity. A. Fetal engagement. A nurse is caring for a client who has been in labor for 12 hr, and her membranes are intact. The provider has decided to perform an amniotomy in an effort to facilitate the progress of labor. The nurse performs a vaginal examination to ensure which of the following prior to the performance of the amniotomy? A. Fetal engagement. B. Fetal lie. C. Fetal attitude. D. Fetal position. C. Rho(D) immune globulin. A nurse is caring for a client who had no prenatal care, is Rh-negative, and will undergo an external version at 37 weeks of gestation. Which of the following medication should the nurse plan to administer prior to the version? A. Prostaglandin gel. B. Magnesium sulfate. C. Rho(D) immune globulin. D. Oxytocin. B. Duration of 90 to 120 seconds. A nurse is caring for a client who is receiving oxytocin for induction of labor and has an intrauterine pressure catheter (IUPC) placed to monitor uterine contractions. For which of the following contraction patterns should the nurse discontinue the infusion of oxytocin? A. Frequency of every 2 min. B. Duration of 90 to 120 seconds. C. Intensity of 60 to 90 mm Hg. D. Resting tone of 15 mm Hg. A. "They are administered in an oral form." A nurse educator in the labor and deliver unit is reviewing the use of chemical agents to promote cervical ripening with a group of newly hired nurses. Which of the following statements by a nurse indicates understudying of the teaching? A. "They are administered in an oral form." B. "They act by absorbing fluid from tissue." C. "The promote dilation of the os." D. "They include an amniotomy." B. Reduced fetal oxygen supply. A nurse is caring for a client who is in lair and experiencing incomplete uterine relaxation between hypertonic contractions. The nurse should identify that this contraction pattern increases the risk for which of the following complications? A. Prolonged labor. B. Reduced fetal oxygen supply. C. Delayed cervical dilation. D. Increased maternal stress. A. Hands and knees. A nurse is caring for a client who is in active labor and reports severe back pain. During assessment, the fetus is noted to be in the occiput posterior positions. Which of the following maternal positions should the nurse suggests to the client to facilitate normal labor progress? A. Hands and knees. B. Lithotomy. C. Trendelenburg. D. Supine with a rolled towel under one hip. D. Prolapsed umbilical cord. A nurse is caring for a client who is admitted to the labor and delivery unit. With the use of Leopold maneuvers, it is noted that the fetus is in a breech presentation. For which of the following possible complications should the nurse observe? A. Preciptitous labor. B. Premature rupture of membranes. C. Postmaturity syndrome. D. Prolapsed umbilical cord. C. Meconium aspiration. A nurse is caring for a client who is at 42 weeks of gestation and in active labor. Which of the following findings is the fetus at risk for developing? A. Intrauterine growth restriction. B. Hyperglycemia. C. Meconium aspiration. D. Polyhydramnios. D. Call for assistance. A nurse is caring for a client in active labor. When last examined 2 hr ago, the client's cervix was 3 cm dilated, 100% effaced, membranes intact and the fetus was at a -2 station. The client suddenly states "My water broke." The monitor reveals a FHR of 80 to 85/min, and the nurse performs a vaginal examination, noticing clear fluid and a pulsing loop of umbilical cord in the client's vagina. Which of the following actions should the nurse perform first? A. Place the client in the trendelenburg position. B. Apply pressure to the presenting part with her fingers. C. Administer oxygen at 10 L/min via a face mask. D. Call for assistance. A. Moderate lochia rubra. A nurse is performing a fundal assessment for a client who is 2 days postpartum and observes there perineal pad for lochia. She notes the pad to be saturated approximately 12 cm with lochia that is bright red and contains small clots. Which of the following findings should the nurse document? A. Moderate lochia rubra. B. Excessive blood loss. C. Light lochia rubra. D. Scant lochia serosa. C. A normal postural discharge of lochia. During ambulation to the bathroom, a postpartum client experiences a gush of dark red blood that soon stops. On assessment, a nurse finds the uterus to be firm, midline, and at the level of the umbilicus. Which of the following findings should the nurse interpret this data as being? A. Evidence of a possible vaginal hematoma. B. An indication of a cervical or perineal laceration. C. A normal postural discharge of lochia. D. Abnormally excessive lochia rubra flow. B. "I need a second vaccination at my postpartum visit." A nurse is completing postpartum discharge teaching to a client who had no immunity to varicella and was given varicella vaccine. Which of the following statements by the client indicates understanding of the teaching? A. "I will need to use contraception for 3 months before considering pregnancy." B. "I need a second vaccination at my postpartum visit." C. "I was given the vaccine because my baby is O-positive." D. "I will be tested in 3 months to see if I have developed immunity." B. Urinary retention. A nurse is assessing a postpartum client for fundal height, location, and consistency. The funds is noted to be displaced laterally to the right, and there is uterine atony. The nurse should identify which of the following conditions as the cause of the uterine atony? A. Poor involution. B. Urinary retention. C. Hemorrhage. D. Infection. A. Change in body fluids. B. Metabolic effort of labor. A nurse is caring for a client who is 1 hr postpartum following a vaginal birth and experiencing uncontrollable shaking. The nurse should understand that the shaking is due to which of the following factors? (SATA) A. Change in body fluids. B. Metabolic effort of labor. C. Diaphoresis. D. Decreases in body temperature. E. Decrease in prolactin levels. D. Provide education about infant care when the father is present. A nurse concludes that the father of an infant is not showing positive signs of parent-infant bonding. He appears very anxious and nervous when the infant's mother asks him to bring her the infant. Which of the following actions should the nurse use to promote father-infant bonding? A. Hand the father the infant, and suggest that he change the diaper. B. Ask the father why he is so anxious and nervous. C. Tell the father that he will grow accustomed to the infant. D. Provide education about infant care when the father is present. B. Give the client time to express her feelings. A client in the early postpartum period is very excited and talkative. She is repeatedly telling the nurse every detail of her labor and birth. Because the client will not stop talking, the nurse is having difficulty completing the postpartum assessments. Which of the following actions should the nurse take? A. Come back later when the client is more cooperative. B. Give the client time to express her feelings. C. Tell the client she needs to be quiet so the assessment can be completed. D. Redirect the client's focus so that she will become quiet. A. Demonstrates apathy when the infant cries. C. Views the infant's behavior as uncooperative during diaper changing. A nurse is caring for a client who is 1 day postpartum. The nurse is assessing for maternal adaptation and mother-infant bonding. Which of the following behaviors by the client indicates a need for the nurse to intervene? (SATA) A. Demonstrates apathy when the infant cries. B. Touches the infant and maintains close physical proximity. C. Views the infant's behavior as uncooperative during diaper changing. D. Identifies and related infant's characteristics to those of family members. E. Interprets the infant's behavior as meaningful and a way of expressing needs. B. "Your son is showing an adverse sibling response." A nurse is caring for a client who is 2 days postpartum. The client states, "My 4-year old son was toilet trained and now he is frequently wetting himself." Which of the following statements should the nurse provide to the client? A. "Your son was probably not ready for toilet training and should wear training pants." B. "Your son is showing an adverse sibling response." C. "Your son may need counseling." D. "You should try sending your son to preschool to resolve the behavior." D. Position the neonate skin-to-skin on the client's chest. A nurse in the delivery room is planning to promote maternal-infant bonding for a client who just delivered. Which of the following is the priority action by the nurse? A. Encourage the parents to touch and explore the neonate's features. B. Limit noise and interruption in the delivery room. C. Place the neonate at the client's breast. D. Position the neonate skin-to-skin on the client's chest. A. "Apply cold compresses between feedings." A nurse is conducting a home visit for a client who is 1 week postpartum and breastfeeding. The client reports breast engorgement. Which of the following recommendations should the nurse make? A. "Apply cold compresses between feedings." B. "Take a warm shower right after feedings." C. "Apply breast milk to the nipples and allow them to air dry." D. "Use the various infant positions for feedings." C. Sore nipple with cracks and fissures. A nurse is providing discharge instructions for a client. At 4 weeks postpartum, the client should contact her provider for which of the following client findings? A. Scant, non odorous white vaginal discharge. B. Uterine cramping during breastfeeding. C. Sore nipple with cracks and fissures. D. Decreased response with sexual activity. A. "Wear a supportive bra continuously for the first 72 hours." A nurse is providing discharge teaching for a non lactating client. Which of the following instructions should the nurse include in the teaching? A. "Wear a supportive bra continuously for the first 72 hours." B. "Pump your breast every 4 hours to relieve discomfort." C. "Use breast shells throughout the day to decrease milk supply." D. "Apply warm compresses until milk suppression occurs." C. Kegel exercises. A nurse is providing discharge instructions to a postpartum client following a cesarean birth. The client reports leaking urine every time she sneezes or coughs. Which of the following interventions should the nurse suggest? A. Sit-ups. B. Pelvic tilt exercise. C. Kegel exercises. D. Abdominal crunches. B. A client who does not wash her hands between perineal care and breastfeeding. A nurse is providing care to four clients on the postpartum unit. Which of the following clients is at greatest risk for developing a postpartum infection? A. A client who has an episiotomy that is erythematous and has extended into a third-degree laceration. B. A client who does not wash her hands between perineal care and breastfeeding. C. A client who is not breastfeeding and is using measures to suppress lactation. D. A client who has a cesarean incision that is well-approximated with no drainage. A. Increasing pulse and decreasing blood pressure. A nurse is caring for a client who is postpartum. The nurse should identify which of the following findings as an early indicator of hypovolemia caused by hemorrhage? A. Increasing pulse and decreasing blood pressure. B. Dizziness and increasing respiratory rate. C. Cool, clammy skin, and pale mucous membranes. D. Altered mental status and level of consciousness. A. Precipitous delivery. C. Inversion of the uterus. E. Retained placental fragments. A nurse educator on the postpartum unit is reviewing risk factors for postpartum hemorrhage with a group of nurses. Which of the following factors should the nurse include in the teaching? (SATA) A. Precipitous delivery. B. Obesity. C. Inversion of the uterus. D. Oligohydramnios. E. Retained placental fragments. A. Calf tenderness to palpation. C. Elevated temperature. D. Area of warmth. A nurse on the postpartum unit is performing a physical assessment of a client who is being admitted with a suspected deep-vein thrombosis (DVT). Which of the following clinical findings should the nurse expect? (SATA) A. Calf tenderness to palpation. B. Mottling of the affected extremity. C. Elevated temperature. D. Area of warmth. E. Report of nausea. D. Measure leg circumferences. A nurse on the postpartum unit is planning care for a client who has thrombophlebitis. Which of the following nursing interventions should the nurse include in the plan of care? A. Apply cold compresses to the affected extremity. B. Massage the affected extremity. C. Allow the client to ambulate. D. Measure leg circumferences. A. Preeclampsia. A nurse is caring for a client who has disseminated intravascular coagulation (DIC). Which of the following antepartum complications should the nurse understand is a risk factor for this condition? A. Preeclampsia. B. Thrombophlebitis. C. Placenta previa. D. Hyperemesis gravidarum. B. A client who had premature rupture of membranes and prolonged labor. A nurse on the postpartum unit is caring for four clients. Which of the following liens should the nurse recognize as the greets risk for development of a postpartum infection? A. A client who experienced a precipitous labor less than 3 hr in duration. B. A client who had premature rupture of membranes and prolonged labor. C. A client who delivered a large for gestational age infant. D. A client who had a boggy uterus that was not well-contracted. C. "Completely empty each breast at each feeding or use a pump." A nurse is teaching a client who is breastfeeding and has mastitis. Which of the following responses should the nurse make? A. "Limit the amount of time the infant nurses on each breast." B. "Nurse the infant only on the unaffected breast until resolved." C. "Completely empty each breast at each feeding or use a pump." D. "Wear a tight-fitting bra until lactation has ceased." B. "I will drink cranberry and prune juices to make my urine more acidic." C. "I will drink large amounts of fluids to flush the bacteria from my urinary tract." E. "I will take Tylenol for any discomfort." A nurse is reviewing discharge teaching with a client who has a urinary tract infection. Which of the following statements by the lucent indicates understanding of the teaching? (SATA) A. "I will perform peri care and apply a perineal pad in a back-to-front direction." B. "I will drink cranberry and prune juices to make my urine more acidic." C. "I will drink large amounts of fluids to flush the bacteria from my urinary tract." D. "I will go back to breastfeeding after I have finished taking the antibiotic." E. "I will take Tylenol for any discomfort." A. Staphylococcus aureus. A nurse is caring for a client who has mastitis. Which of the following is the typical causative agent of mastitis? A. Staphylococcus aureus. B. Chlamydia trachomatis. C. Klebsiella pneumonia. D. Clostridium perfringens. A. Epidural anesthesia. B. Urinary bladder catheterization. C. Frequent pelvic examinations. D. History of UTIs. A nurse is discussing risk factors for urinary tract infections with a newly licensed nurse. Which of the following conditions should the nurse include in the teaching? (SATA) A. Epidural anesthesia. B. Urinary bladder catheterization. C. Frequent pelvic examinations. D. History of UTIs. E. Vaginal birth. D. Postpartum blues. A nurse is assessing a postpartum client who is exhibiting tearfulness, insomnia, lack of appetite, and a feeling of letdown. Which of the following conditions are associated with these clinical findings? A. Postpartum fatigue. B. Postpartum psychosis. C. Letting-go phase. D. Postpartum blues. A. Fatigue. B. Insomnia. D. Flat affect. A nurse is caring for a postpartum client who delivered her third infant 2 days ago. The nurse recognizes that which of the following findings are suggestive of postpartum depression? (SATA) A. Fatigue. B. Insomnia. C. Euphoria. D. Flat affect. E. Delusions. B. Concerns about lack of income to pay bills. C. Anxiety about assuming a new role as a mother. D. Rapid decline in estrogen and progesterone. E. Feeling of inadequacy with the new role as a mother. A nurse is assessing a client who has postpartum depression. The nurse should expect which of the following findings? (SATA) A. Paranoia that her infant will be harmed. B. Concerns about lack of income to pay bills. C. Anxiety about assuming a new role as a mother. D. Rapid decline in estrogen and progesterone. E. Feeling of inadequacy with the new role as a mother. B. Ask the client if she has thoughts of harming herself or her infant. A nurse is caring for a client who has postpartum psychosis. Which of the following actions is the nurse's priority? A. Reinforce the need to take antipsychotics as prescribed. B. Ask the client if she has thoughts of harming herself or her infant. C. Monitor the infant for indications of failure to thrive. D. Review the client's medical record for a history of bipolar disorder. B. Appropriate for gestational age. A nurse is caring for a newborn who was born at 38 weeks of gestation, weighs 3,200 g, and is in the 60th percentile for weight. Based on the weight and gestational age, the nurse should classify this neonate as which of the following? A. Low birth weight. B. Appropriate for gestational age. C. Small for gestational age. D. Large for gestational age. D. Epstein's pearls. A nurse is completing a newborn assessment and observes small white nodules on the roof of the newborn's mouth. This finding is a characteristic of which of the following conditions? A. Mongolian spots. B. Milia spots. C. Erythema toxicum. D. Epstein's pearls. D. Hold the newborn in a semi-sitting position, then allow the newborn's head and trunk to fall backward. A nurse is assessing the reflexes of a newborn. In checking for the Moro reflex, the nurse should perform which of the following? A. Hold the newborn vertically under arms and allow one foot to touch table. B. Stimulate the pads of the newborn's hands with stroking or massage. C. Stimulate the soles of the newborn's feet not the outer lateral surface of each foot. D. Hold the newborn in a semi-sitting position, then allow the newborn's head and trunk to fall backward. C. Apnea for 10-second periods. D. Obligatory nose breathing. A nurse is completing an assessment. Which of the following data indicates the newborn is adapting to extrauterine life? (SATA) A. Expiratory grunting. B. Inspiratory nasal flaring. C. Apnea for 10-second periods. D. Obligatory nose breathing. E. Crackles and wheezing. A. "This is frequently seen in newborns who have dark skin." A nurse is teaching a newly licensed nurse how to bathe a newborn and observes a bluish marking across the newborn's lower back. The nurse should include which of the following information in the teaching? A. "This is frequently seen in newborns who have dark skin." B. "This is a finding indicating hyperbilirubinemia." C. "This is a forceps mark from an operative delivery." D. "This is related to prolonged birth or trauma during delivery." C. Erythromycin. A nurse is preparing to administer prophylactic eye ointment to a newborn to prevent opthalmia neonatorum. Which of the following medications should the nurse anticipate administering? A. Ofloxacin. B. Nystatin. C. Erythromycin. D. Ceftriaxone. C. Evaporation. A newborn was not dried completely after birth. Which of the following mechanisms should the nurse understand causes heat loss? A. Conduction. B. Convection. C. Evaporation. D. Radiation. D. Covering the newborn's head with a cap. A nurse is caring for a newborn immediately following birth. Which of the following nursing interventions is the highest priority? A. Initiating breastfeeding. B. Performing the initial bath. C. Giving a vitamin K injection. D. Covering the newborn's head with a cap. A. "It assists with blood clotting." A nurse is preparing to administer a vitamin K (phytonadione) injection to a newborn. Which of the following responses should the nurse make to the newborn's mother regarding why this medication is given? A. "It assists with blood clotting." B. "It promotes maturation of the bowel." C. "It is a preventative vaccine." D. "It provides immunity." C. Match the mother's identification band with the newborn's band. A nurse is taking a newborn to a mother following a circumcision. Which of the following actions should the nurse take for security purposes? A. Ask the mother to state her full name. B. Look at the name on the newborn's bassinet. C. Match the mother's identification band with the newborn's band. D. Compare name on the bassinet and room number. D. When latched on, the infant's nose, cheek, and chin are touching the breast. A nurse is giving instructions to a mother about how to breastfeed her newborn. Which of the following actions by the mother indicates understanding of the teaching? A. The mother places a few drops of water on her nipple before feeding. B. The mother gently removes her nipple from the infant's mouth to break the suction. C. When she is ready to breastfeed, the mother gently strokes the newborn's neck with her finger. D. When latched on, the infant's nose, cheek, and chin are touching the breast. C. Keep the nipple full of formula throughout the feeding. A nurse is teaching a group of new parents about proper techniques for bottle feeding. Which go the following instructions should the nurse provide? A. Burp the newborn at the end of the feeding. B. Hold the newborn close in a supine position. C. Keep the nipple full of formula throughout the feeding. D. Refrigerate any unused formula. B. Attempts to place his hand in his mouth. A nurse is caring for a newborn. Which of the following actions by the newborn indicates readiness to feed? A. Spits up clear mucus. B. Attempts to place his hand in his mouth. C. Turns his head toward sounds. D. Lies quietly with his eyes open. C. Place used bottles in the dishwasher. D. Check the nipple for appropriate flow of formula. E. Use tap water to dilute concentrated formula. A nurse is reviewing formula preparation with parents who plan to bottle-feed their newborn. Which of the following information should the nurse include in the teaching? (SATA) A. Use a disinfectant wipe to clean the life of the formula can. B. Store prepared formal in the refrigerator for up to 72 hr. C. Place used bottles in the dishwasher. D. Check the nipple for appropriate flow of formula. E. Use tap water to dilute concentrated formula. D. Cradle. A nurse is reviewing breastfeeding positions with the mother of a newborn. Which of the following positions should the nurse discuss? A. Over-the-shoulder. B. Supine. C. Chin-supported. D. Cradle. D. Keep the diaper folded below the cord. A nurse is reviewing care of the umbilical cord with the parent of a newborn. Which of the following instructions should the nurse include in the teaching? A. Cover the cord with a small gauze square. B. Trickle clean water over the cord with each diaper change. C. Apply hydrogen peroxide to the cord twice a day. D. Keep the diaper folded below the cord. A. Hypospadias. C. Family history of hemophilia. E. Epispadias. A nurse is reviewing contraindications for circumcision with a newly hired nurse. Which of the following conditions are contraindications? (SATA) A. Hypospadias. B. Hydrocele. C. Family history of hemophilia. D. Hyperbilirubinemia. E. Epispadias. C. "I will clean his penis with each diaper change." A nurse is providing discharge teaching to the parents of a newborn regarding circumcision care. Which of the following statements made by a parent indicates an understanding of the teaching? A. "His circumcision will heal within a couple of days." B. "I should remove the yellow mucus that will form." C. "I will clean his penis with each diaper change." D. "I will give him a tub bath within a couple of days." C. Apply petroleum gauze to the site. A nurse is caring for a newborn immediately following a circumcision using a Gomco procedure. Which for the following actions should the nurse implement? A. Apply Gelfoam powder to the site. B. Place the newborn in the prone position. C. Apply petroleum gauze to the site. D. Avoid changing the diaper until the first voiding. C. Back seat, rear-facing. A nurse is reviewing car seat safety with the parents of a newborn. Which of the following instructions should the nurse include in the teaching regarding car seat position? A. Front seat, rear-facing. B. Front seat, forward-facing. C. Back seat, rear-facing. D. Back seat, forward-facing. D. "Your baby's skin will have a leathery appearance." A nurse is caring for a client who is at 42 weeks gestation and in labor. The client asks the nurse what should she expect because her baby is postmature. Which of the following statements should the nurse make? A. "Your baby will have excess body fat." B. "Your baby will have flat areola without breast buds." C. "Your baby's heels will easily move to his ears." D. "Your baby's skin will have a leathery appearance." C. Sunken fontanels. A nurse is caring for an infant who has a high bilirubin level and is receiving phototherapy. Which of the following is the priority finding in the newborn? A. Conjunctivitis. B. Bronze skin discoloration. C. Sunken fontanels. D. Maculopapular skin rash. A. Lanugo. C. Weak grasp reflex. D. Translucent skin. A nurse is called to the birthing room to assist with the assessment of a newborn who was born at 32 weeks of gestation. The newborn's birth weight is 1,100 g. Which of the following are expected findings in this newborn? (SATA) A. Lanugo. B. Long nails. C. Weak grasp reflex. D. Translucent skin. E. Plump face. A. Oxygen saturation. A nurse is caring for a newborn who is preterm and has respiratory distress syndrome. Which of the following should the nurse monitor to evaluate the newborn's conditions following administration of synthetic surfactant? A. Oxygen saturation. B. Body temperature. C. Serum bilirubin. D. Heart rate. B. "The newborn will have a continuous hight-pitched cry." A nurse is teaching a newly licensed nurse about neonatal abstinence syndrome. Which of the following statements by the newly licensed nurse indicate understanding of the teaching? A. "The newborn will have decreased muscle tone." B. "The newborn will have a continuous hight-pitched cry." C. "The newborn will sleep for 2 to 3 hours after a feeding." D. "The newborn will have mild tremors when disturbed." A nurse in a prenatal clinic is caring for a client who believes that she might be pregnant because she feels the baby moving. Which of the following statements should the nurse make? A. "This is a presumptive sign of pregnancy." B. "This is a probable sign of pregnancy." C. "This is a possible sign of pregnancy." D. "This is a positive sign of pregnancy." A. Presumptive signs of pregnancy include physical changes that are apparent to the client, such as quickening. A nurse in a prenatal clinic is caring for a client who is at 12 weeks gestation. The client asks about the cause of her heartburn. Which of the following responses should the nurse make? A. Retained bile in the liver results in delayed digestion. B. Increased estrogen production causes increased secretion of hydrochloric acid. C. Pressure from the growing uterus displaces the stomach. D. Increased progesterone production causes decreased motility of smooth muscle. D. Increased progesterone production causes a relaxation of the cardiac sphincter of the stomach and delayed gastric emptying, which can result in heartburn. A nurse is caring for an antepartum client whose laboratory findings indicate a negative rubella titer. Which of the following is the correct interpretation of this data? A. The client is not experiencing a rubella infection at this time. B. The client is immune to the rubella virus. C. The client requires a rubella vaccination at this time. D. The client requires a rubella immunization following delivery. D. A negative rubella titer indicates that the client is susceptible to the rubella virus and needs vaccination following delivery. Immunization during pregnancy is contraindicated because of possible injury to the developing fetus. Following rubella immunization, the client should be cautioned not to conceive for 1 month. A nurse in a prenatal clinic is caring for a client who is pregnant and asks the nurse for her estimated date of birth (EDB). The client's last menstrual period began on July 27. What is the client's EDB? (State the date in MMDD. For example, July 27 is 0727). 0504. Using Nägele's rule, the nurse subtracts three months from the date of the last menstrual period, then adds 7 days. July minus 3 months equals April. There are 30 days in April, so 27 + 7 = May 4. The client's EDB is May 4, which would be written as 0504 in the MMDD format. A nurse is caring for a client at the first prenatal visit who has a BMI of 26.5. The client asks how much weight she should gain during pregnancy. Which of the following responses should the nurse make? A. "It would be best if you gained about 11 to 20 pounds." B. "The recommendation for you is about 15 to 25 pounds." C. "A gain of about 25 to 35 pounds is recommended for you." D. "A gain of about 1 pound per week is the best pattern for you." B. Clients who are overweight, having a BMI of 25 to 29.9, should be advised that the recommended weight gain is 7 to 11.5 kg (15 to 25 lb). The pattern of weight gain is also important, with minimal gain in the first trimester. A nurse is providing teaching to a client who is planning on becoming pregnant about the changes she should expect. Identify the sequence of maternal changes. (Move the steps into the box on the right, placing them in the selected order of performance. Use all the steps.) -Quickening -Lightening -Goodell's sign -Amenorrhea Amenorrhea, a presumptive sign of pregnancy, is one of the first physiological indications of pregnancy that occurs by 4 weeks of gestation. Goodell's sign, a probable sign of pregnancy, is the next of physiological indications to occur. Goodell's sign is the softening of the cervix that typically occurs at 5 to 6 weeks of gestation. Quickening, the mother's perception of the first fetal movement, is a presumptive sign of pregnancy that typically occurs between 16 and 20 weeks of gestation. Lightening is the last of these physiological signs of pregnancy to occur. As the fetus descends into the pelvic cavity the fundal height decreases, which typically occurs between 38 and 40 weeks of gestation. A nurse is teaching a client who is at 23 weeks of gestation about immunizations. Which of the following statements should the nurse include in the teaching? A. "You should not receive the rubella vaccine while breastfeeding." B. "You should receive a vaccine before you deliver." C. "You can receive an influenza vaccination during pregnancy." D. "You cannot receive the Tdap vaccine until after you deliver." C. It is recommended that pregnant women receive annual influenza vaccinations. A nurse is providing education to a client during the first prenatal visit. Which of the following statements by the client should indicate to the nurse a need for clarification? A. "I should drink about 2 liters of fluid each day." B. "It is okay for me to eat cold cuts so long as they are heated until steaming hot just before serving." C. "I can have a moderate amount of caffeine daily." D. "I should increase my calcium intake to 1,500 milligrams per day" D. A woman's dietary reference intake (DRI) of calcium for pregnancy and lactation is the same for a woman who is not pregnant. The DRI for a woman older than 19 years of age is 1,000 mg/day, which should supply enough calcium for fetal bone and tooth development and to maintain maternal bone mass. A nurse at a prenatal clinic is caring for a client who is in her first trimester of pregnancy. The client tells the nurse that she is upset because, although she and her husband planned this pregnancy, she has been having many doubts and second thoughts about the upcoming changes in her life. Which of the following is an appropriate response by the nurse? A. "Ambivalent feelings are quite common for women early in pregnancy." B. "Perhaps you should see a counselor to discuss these feelings further." C. "Have you spoken to your mother about these feelings?" D. "Don't worry. You will be fine once the baby is born." A. This response uses the therapeutic communication technique of providing information while addressing the client's concerns and feelings. This statement is true and gives the client the information she needs; many antepartum women experience similar feelings in early pregnancy. The nurse is assessing the laboratory report of a 40-week gestation client. Which of the following values would the nurse expect to find elevated above prepregnancy levels? A. glucose B. fibrinogen C. hematocrit D. bilirubin B. A nurse in a prenatal clinic is completing a skin assessment of a client who is in the second trimester. Which of the following findings should the nurse expect? (Select all that apply). A. Eczema B. Psoriasis C. Linea nigra D. Chloasma E. Striae gravidarum C, D, E. Eczema is incorrect. Eczema manifests as red, swollen, and itchy skin and is not an expected finding during pregnancy. Psoriasis is incorrect. Psoriasis manifests as thick red patches or plagues covered by silver scales on the skin and is not an expected finding during pregnancy. Linea nigra is correct. Linea nigra manifests as a line of pigmentation extending from the symphysis pubis to the top of the fundus and is an expected finding during pregnancy. Chloasma is correct. Chloasma, or the mask of pregnancy, manifests as blotchy, brownish hyperpigmentation of the skin over the forehead, nose, and cheeks and is an expected finding during pregnancy. Striae gravidarum is correct. Striae gravidarum, or stretch marks, occur because of the separation of underlying connective tissue on the breasts, thighs, and abdomen. They are an expected finding during pregnancy. A nurse is caring for a client who has a positive pregnancy test. The nurse is teaching the client about common discomforts in the first trimester of pregnancy as well as warning signs of potential danger. The nurse should instruct the client to call the clinic if she experiences which of the following manifestations? A. Leukorrhea B. Urinary frequency C. Nausea and vomiting D. Facial edema D. Facial edema is a warning sign of a hypertensive condition or preeclampsia and should be reported immediately to the provider. A nurse in a prenatal clinic is caring for a client who is at 7 weeks of gestation. The client reports urinary frequency and asks if this will continue until delivery. Which of the following responses should the nurse make? A. "It's a minor inconvenience, which you should ignore." B. "In most cases it only lasts until the 12th week, but it will continue if you have poor bladder tone." C. "There is no way to predict how long it will last in each individual client." D. "It occurs during the first trimester and near the end of the pregnancy." D. Urinary frequency is due to increased bladder sensitivity during the first trimester and recurs near the end of the pregnancy as the enlarging uterus places pressure on the bladder. A nurse is providing teaching about Kegel exercises to a group of clients who are in the third trimester of pregnancy. Which of the following statements by a client indicates understanding of the teaching? A. "These exercises help prevent constipation." B. "These exercises help pelvic muscles to stretch during birth." C. "They can help reduce back aches." D. "They can prevent further stretch marks." B. Kegel exercises improve the strength of perineal muscles, facilitating stretching and contracting during childbirth. A nurse is providing teaching about nutrition to a client at her first prenatal visit. Which of the following statements by the nurse should be included in the teaching? A. "You will need to increase your calcium intake during breast feeding." B. "Prenatal vitamins will meet your need for increased vitamin D during pregnancy." C. "Vitamin E requirements decline during pregnancy due to the increase in body fat." D. "You will need to double your intake of iron during pregnancy." D. During pregnancy, the need for iron increases to allow transfer of the appropriate amounts to the fetus and to support expansion of the client's red blood cell volume. A nurse is assessing a client who is in the third trimester of pregnancy. The nurse should recognize which of the following findings as an expected physiologic change during pregnancy? A. Gradual lordosis B. Increased abdominal muscle tone C. Posterior neck flexion D. Decreased mobility of pelvic joints A. Clients who are pregnant can develop a gradual, forward curving of the spine as the growth of the fetus pulls the pelvis forward. This lordosis resolves after delivery. A nurse is caring for a client who is at 18 weeks of gestation. The client tells the nurse that she felt fluttering movements in her abdomen 3 days ago. The nurse should interpret this finding as which of the following? A. Ballottement B. Lightening C. Quickening D. Chloasma C. Clients describe quickening as a fluttering sensation, which can be felt as early as the 14th week of gestation although is most typically felt between 16-20 weeks gestation (closer to 16 for multipara and closer to 20 for primipara patients). It reflects fetal movement. A nurse is caring for a client who is in her first trimester of pregnancy and asks the nurse if she can continue to exercise during pregnancy. Which of the following responses by the nurse is appropriate? A. "Exercising during pregnancy is not recommended." B. "Daily jogging for up to 30 minutes is fine throughout the pregnancy." C. "Activities that raise the body temperature, such as saunas and hot tubs, are safe until the third trimester." D. "It is recommended that pregnant clients limit their exercise routine to stretching activities on a mat several times a week." B. While weight-bearing exercises might become uncomfortable in the last trimester, they are generally not contraindicated, providing the client stays hydrated and avoids becoming overheated for extended periods. A nurse in a prenatal clinic is reviewing the health record of a client who is at 28 weeks of gestation. The history includes one pregnancy, terminated by elective abortion at 9 weeks; the birth of twins at 36 weeks; and a spontaneous abortion at 15 weeks. According to the GTPAL system, which of the following describes the client's current status? A. 4-0-1-2-2 B. 3-0-2-0-2 C. 2-0-0-2-0 D. 4-2-0-2-2 A. This response correctly describes the client's current status: pregnant currently and had 3 prior pregnancies (G); no term births (T); one pregnancy resulted in the preterm birth (P) of twins; two pregnancies ended in abortion (A); and she has two living children (L). A nurse is preparing to measure the fundal height of a client who is at 22 weeks of gestation. At which location should the nurse expect to palpate the fundus? A. 5 cm above the umbilicus B. Slightly above the umbilicus C. Slightly below the umbilicus D. 3 cm below the umbilicus B. At 22 weeks of gestation, the fundal height should be just above the level of the umbilicus. The distance in centimeters from the symphysis pubis to the top of the fundus is a gross estimate of the weeks of gestation. Terms in this set (100) A nurse is caring for a client who has a suspected ectopic pregnancy at 8 weeks gest. Which of the following manifestations should the RN ID as consistent with the diagnosis unilateral, cramp-like abdominal pain A nurse in a clinic is assessing a client who is 8 wks gest and has HG. findings to expect? hx of migraines - first 20 wks nulliparous - first trimester twin gestation - increasing levels of estrogen, progesterone, HCG teaching fetal development to group of clients at antenatal clinic, nurse include in the teaching? the baby's heart beat is audible by a doppler stethoscope at 12 weeks of preg sex is determined at conception lanugo covers fetal body at 20wks and disappears by week 36 quickening the feeling of fetal movement begins between weeks 14 to 16 in multiparous and week 18 or later in nulliparous assessing client at 3 days PP and is breastfeeding. then nurse notes that the fundus is three fingerbreadths below the umbilicus, lochia rubra is moderate, and the breasts are hard and warm on palpation. interpretations of these findings? additional intervention is not indicated at this time. the fundal location and lochia are characteristics w/in normal range. breast engorgement is typical, breast begin to produce milk. frequent BF and routine care can help relieve engorgement. nurse in a prenatal clinic is reviewing client record who is 28 weeks of gest. the hx includes 1 preg, terminated at 9 weeks, birth of twins at 36 weeks, abortion at 15 weeks. What is GTPAL? G 4 T 0 P 1 A 2 L 2 nurse is teaching client PP and has new RX for injection of RHOGAM, what should be included in teaching? it prevents formation of RH antibodies in mothers who are RH negative given at 28week and following future preg Only $1/month teaching a newborn parent to care for the umbilical cord stump. include which instructions? give sponge bath till cord falls off caring for newborn, grandmother of the newborn asks if she can take the baby to the mothers room. Appropriate response? have the mother call and I will take the baby to the room. a nurse assessing a client who is 8hr PP and multiparous. Which of the following findings should alert the nurse to the client's need to urinate? fundus is three fingerbreadths above the umbilicus a nurse in the ED is admitting a pt who is 40 wks gest, has ruptured membranes, nurse observes NB head crowning. Client tells nurse she wants to push. Correct reply? you should try to pant as the delivery proceeds panting allows uterine forces to expel the fetus and permits controlled muscle expansion to avoid rapid expulsion of fetal head a nurse is completing HX for client at 6wks gest. Client informs the nurse that she smokes one pack of cig a day. the nurse should advise the client that smoking places the clients NB at risk for...? IUGR - intrauterine growth restriction placental abruption placenta previa preterm delivery fetal death a nurse is caring for a client who is PP. nurse should recognize the following as indication of inhibition of parenteral attachment? i wish he had more hair, i will keep a hat on his head till he grow some a nurse caring for a client who delivered a healthy NB via C-section birth. client asks the nurse, is there a chance that i could deliver my next baby w/out have a c-section. best response? the primary consideration is what type of incision was performed this time. a nurse is preparing to administer MgSO4 IV to client, who is experiencing preterm labor. priority nursing assessment for client? RR nurse is caring for a client who is primigravida at term, and having contractions, but stating that she is not really sure if she is in labor or not. nurse should recognize as a sign of true labor? change in cervix nurse is planning care for a NB who is SGA, which is priority intervention to include in care? monitor blood glucose nurse is caring for a client 2 days PP, is BF, and reports nipple soreness. Which measures would reduce discomfort during BF? apply BM to nipples before feeding start on less sore nipple change infant position on nipple A nurse is s office is caring for a client who is 34 weeks and at risk for placental abruption. nurse should recognize which common risk factor for abruption? HTN nurse in the ambulatory surgery center is providing discharge teaching to client who had a D&C, following a spontenous miscarriage. include in the teaching? vaginal intercourse can be resumed after 2 weeks patient at 38 weeks, large amount of painless bright red vaginal bleeding. placed on fetal monitor; indicating FHR of 138/min, no uterine contractions. client vital signs are BP: 98/52, HR 118/min, RR 24/min, Temp 36.4 - 97.6,. priority action? initiate IV access - large-bore IV catheter client is losing blood rapidly, HYPOTENSION - TACHYCARDIA - NEED fluids and blood to be administered IF HYPOVOLEMIA develops nurse is caring for client, first trimester, ask nurse if she can continue to exercise during pregnancy. Appropriate response? daily jogging for 30/min/day is fine throughout preg nurse is caring for a new mother, concerned her NBs eyes cross. therapeutic response? this occurs because NBs lack muscle control to regulate eye movement nurse in prenatal clinic is caring for client, first trimester, client tells nurse she is upset, planned pregnancy, but having doubts and second thoughts. Appropriate response? ambivalent feelings are quite common for women early in preg nurse on L&D unit, client having difficult, prolonged labor with severe backache, contributing causes should the nurse ID? fetal position is persistent Occiput Posterior nurse is caring for client, 1hr PP, observes large amount of lochia rubra, several small clots on pad. fundus is midline and firm at the umbilicus. nurse action to take? document findings and continue to monitor the client. expected findings. nurse is caring for client, 1st stage of labor, observes umbilical cord protruding from the vagina. first action to perform? insert gloved hand into vagina to relieve pressure on the cord nurse in prenatal clinic, caring for client at 7 weeks. client reports urinary frequency, asks if this will continue after delivery. appropriate response? it occurs during the first trimester and near the end of preg nurse in prenatal clinic, caring for client suspected of having a hydatidform mole. findings to expect? excessive uterine enlargement nurse is completing home visit to mother, 3 days PP, BF, mother expresses concern the amount of weight the newborn lost since birth. response by nurse? this might be related to your baby having 3 stools a day. the NB is being BF typically has 3 or more stools per day during the first few weeks. a nurse in L&D is caring for client undergoing EFM, nurse observes the FHR begins to slow after the start of contraction. the lowest rate occurs after the peak of the contraction. first action by nurse? place the client in the lateral position, LATE DECEL, fetal hypoxemia due to insufficient placental perfusion. nurse is caring for a preterm newborn who is in an incubator to maintain neural thermal environment. the father of the NB asks the nurse why is this necessary. response? preterm NB lack adequate temperature control mechanisms nurse on L& D unit is caring for client following vaginal exam, by provider which documents, -1, interpret? the present part is 1 cm above ischial spine a nurse is providing teaching about KEGELs to group of clients who are in third trimester. indicates understanding of teaching? these exercises help pelvic muscles to stretch during birth nurse in prenatal clinic is teaching client, new prescription for dinoprostone gel. nurse include in the teaching? softens cervix stimulates uterine muscles nurse caring for client who is in 1st stage of labor, using pattern-paced breathing. client states she feels lightheaded, fingers are tingling. nurse should ? assist client to breathe into a paper bag nurse caring for client, considering several methods of contraception, ID as being most reliable? IUD -intrauterine device nurse providing teaching to client about nutrition, at first prenatal visit. include in teaching? you will need to double your intake of iron during pregnancy nurse is instructing woman, contemplating pregnancy about nutritional needs. to reduce risk of giving birth to NB w/ neural tube defect, include in teaching? consume foods fortified with folic acid nurse is completing discharge instructions for new mother and 2 day old NB. mother asks, how will i know if my NB gets enough BF? appropriate response? your baby should have 6-9 wet diapers a day nurse caring for adolescent PT who has PID as consequence of sexually transmitted infection, will need IV antibiotic therapy. the client tells the nurse, my parents think i am a virgin, I don't think i can tell them i have this kind of infection. appropriate response? you seemed scared to talk to your parents nurse is caring for client in prenatal visit, BMI is 26.5, client ask how much weight she should gain during preg. appropriate response? the recommendation for you is 15-25lbs BMI <18.5 - UNDERWT. 18 - 24.9 - NORMAL 25 - 29.9 OVERWT. >30 - OBESE nurse caring for client, scheduled for a maternal serum alpha fetoprotein test at 15 wks. explanation about test to client? SCREENING test for spinal defects in fetus neural tube defects abdominal wall defects microcephaly anencepaly basis for possible further testing - Aminiocentesis and specialized ultrasounds nurse is caring for client who is in preterm labor at 32 weeks . The client asks will my baby be okay? appropriate response? you must be feeling scared and powerless nurse caring for client beginning BF after delivery, new mother states, i don't want to take anything for pain because i am BF, appropriate response? we can time your medication so that you have an hour or two before next feeding nurse is caring for an antepartum client, lab findings indicate rubella titer. correct interpretation of this data? requires a rubella immunization following delivery nurse caring for client , just delivered NB, following delivery, nurse action first to care for NB? clear the Resp tract nurse is prenatal clinic is caring for client, believes she might be pregnant, she feels the baby moving, appropriate statement? presumptive sign nurse in prenatal clinic caring for client, pregnant, asks nurse of her EDB, LMP began july 27, what is EDB? 05 04 30 days in april + 7 days = may 4 SUBTRACT 3 MONTHS FROM LMP month ADD 7 DAYS nurse caring for client, PP, asks when her BM will come on, appropriate response? in 3-5 days nurse caring for adolescent who is G1P0, admitted to hospital at 38wks with dx of preeclampsia, findings that are inconsistent with preeclampsia DTR - +1 should be increased. OTHER - expected findings 1+ pitting edema +3 proteinuria in urine BP 148/98 nurse is caring for client, 1st stage of labor, undergoing external FHR and recieving IV fluids. nurse observes variable decel in the FHR on the monitor strip. correct interpretation ? variable decel is due to umbilical cord compression nurse in prenatal clinic, caring for client, using leopolds manuvers, palpates a round, firm, moveable part in the fundus of the tuerus and a long smoth surface on the clients right side. which quadrant should the nurse expect to auscultate FHT right upper, breech presentation nurse caring for pt in labor, which of the following reflects application of the gate control theory of pain? massage the clients back theory is based on concept of blocking or preventing the transmission of pain signals to the brain by using distraction techniques nurse in community clinic, counseling client who received positive test result for chlamydia. appropriate statement? this infection is treated with one dose of azithromycin nurse is preparing to admi methylergonovine IM to client who experienced vaginal delivery . nurse should explain to client, purpose of this medication to prevent, condition? PPH a nurse is preparing to assess a newborn who is postmature. findings to expect? cracked peeling skin positive moro reflex nurse is leading discussion about contraception with group of 14 y.o clients. after presentation, client ask which method would be best for her to use. appropriate response? before i can help you, i need to know more about your sexual activity nurse in L&D, admitting a client who reports painful contractions, nurse determine contraction duration of 1min and frequency of 3 min. the nurse obtains VS, FHR - 130/min, MHR 128/min, MBP: 92/54. PRIORITY action position pt with one hip elevated - client needs adequate BP to perfuse herself and her fetus BP 92/54 nurse is reviewing the hx of client who has new prescription of combined OC. nurse recognizes which medications can interfere with the effectiveness of OC? anticonvulsants nurse in prenatal clinic, caring for client at 38wks, reports heavy red vaginal bleeding. the bleeding started spontaneously in the morning, not accompanied by contractions. client is not in distressed, she states she can feel the baby moving. an ultrasound is scheduled STAT. explain to client the purpose of ultrasound to determine? location of placenta - placenta previa nurse is caring for client who is positive pregnancy test, nurse is teaching client about common discomfort in 1st trimester of pregnancy and warning signs danger. instruct client to call clinic if she experiences manifestations? facial edema - HTN or preeclampsia home heatlh nurse is teaching client, BF managing breast engorement. indicates understanding of teaching? i'll feed the baby every 2 hrs nurse is caring for client w/ preeclampsia and is being tx with MgSO4 IV, clients RR is 10/min. DTR absent. priority action? D/C medication infusion nurse in prenatal clinic is caring for a client who asks what is her EDD, LMP May 4, 2015, appropriate response? Feb 11 2016 count 9months ahead plus 7 days nurse is caring for client who is 39 weeks and is in active labor, nurse locates the fetal heart tones above the umbilicus at midline. should suspect fetus is in which position? frank breech nurse is caring for client who is having a NST performed. the FHR is 130-150/min, but there is no fetal movement for 15 min. nurse action? offer a snack of OJ and crackers, baby is probably sleeping. turning to lateral side, DOES NOT indicate FHR is an issue nurse is caring for a client, 40 wks and in labor. ultrasound examination indicates that fetus is SGA. intervention to include for NB care? observe for meconium in Respiratory secretions SGA= increase risk of INTRAUTERINE HYPOXIA due to the presence of meconium in aminiotic fluid at risk for perinatal asphyxia due to stress of labor and depressed nurse is caring for pt, BF, asks the nurse about changes to make in her diet. dietary changes nurse suggest? reduce intake of IRON nurse is preparing to measure fundal height of client who is 22 wks, at which location should the nurse palpate the fundus?? slightly above umbilicus a nurse is caring for client, 36 wk, suspected of placenta previa, findings support DX?? painless red vaginal bleeding nurse is caring for a client who is PP, recieved methylergonovine. findings indicate the medication was effective?? fundus is firm to palpate nurse caring for pt, 6 wks, first preg, asks nurse when she can expect to exp quickening, appropriate response? occur between the fourth and fifth month nurse is assessing NB immediately following c-section delivery, following assessment priority?? respiratory distress nurse is admitting pt who has severe preeclampsia, 35 wks, review providers orders, which needs clarification? question order for ambulate 2x daily nurse is prenatal clinic caring for client, 12 wks, asks about cause of her heartburn, nurse response? increased progesterone production causes decreased motility of smooth muscle nurse is caring for client at 40wks, in active labor, 6cm, 100%,, obtained pt BP 82/52. nursing intervention?? assist client to turn onto her side a nurse is instructing a female client about check BASAL TEMP to determine client is ovulating. nurse instruct client to check her temp at what time? early in the morning before arising nurse is completing admission assessment of a newborn, following anatomical landmarks, the nurse can use which to measure NB chest circumference?? nipple line nurse is caring for client who experienced a vaginal delivery 12 hr ago. when palpating the clients abdomen, which position should the nurse expect to find the uterine fundus? at the level of the umbilicus nurse is caring for several clients. the nurse should recognize that it is safe to administer tocolytic therapy to which client? a client who is experiencing preterm labor at 26 wks nurse is assessing a client in active labor, and notes the presenting part is 0 station. correct interpretation of clinical finding? the lowermost portion of the fetus is at the level of the ischial spines nurse is caring for a client PP, client tells nurse that the NB maternal grandma was born deaf, and asks how to tell if the NB hears well. correct statement to make? we do routine hearing screenings on NBS you'll know the results before you leave the hospital nurse is caring for client who has rubella at the time of delivery and ask why her NB is being placed in isolation. response? the NB might be actively shedding the virus nurse is a college health clinic is speaking to group of adolescents about TSS, include in teaching about increasing risk of contracting TSS? high absorbing tampons nurse is completing discharge teaching to a client , 35th wek, has mild preeclampsia. information about nutrition should be in teaching? drink 48 to 64 oz of water daily nurse in family planning clinic, caring for 17y.o, request OC, client states she is nervous, never had a pelvic exam, nurse response? what part of the exam makes you most nervous? nurse assessing NB 1 hr after birth, RR w/in expected range? 48/ min 30-60bpm nurse is caring for pt during 1st trimester, after reviewing pt blood work, nurse notices she does not have immunity to rubella, which times should the nurse understand is recommended for rubella immunization? shortly after birth nurse in prenatal clinic, completing skin assessment for pt in 2nd trimester. expected findings? linea nigra chloasma striae gravidarum nurse providing preconception counseling for pt who is planning preg, which supplement should the nurse recommend to prevent neural tube defect in fetus? folic acid nurse is caring for a client who is at 18wk, client tells nurse she felt fluttering movements in her abdomen 3 days ago, interpret findings as? quickening around 14th week nurse is caring for client who is to undergo amniotomy, priority action?? assess the FHR pattern nurse is caring for client in labor, EFmonitor. observe late decels, on the strip and interprets this as? uteroplacental insuffficiency a nurse is assessing a NB at 42.5 wks, expect findings? dry cracked skin nurse is caring for client who is premature labor, recieving terbutaline, nurse should monitor client for what A/E to report to provider? dyspnea - pulmonary edema nurse assisting w/ BF, explains which reflex will promote NB latch? rooting nurse is caring for client who is to undergo, BPP, the client asks nurse what is being evaluated, nurse should include? fetal breathing fetal motion aminiotic fluid nurse cairng for NB immediately after birth, after assuring patent airway, what is priority?? dry the skin nurse caring for NB, who has myleomeningocele, nurse goal priority in care for NB? myelomeningocele is a congenital d/o that causes the spine and spinal canal to not close prior to birth, which results in the spinal cord, meninges, and nerve roots, protruding out of the childs back, in the fluid filled sac. before surgery NB must be held carefully to reduce damange to the exposed spinal cord and. maintain integrity of the sac nurse in labor and delivery, pt following delivery of placenta, the nurse examines the umbilical cord, expected findings to observe on cord? two ARTERIES one VEIN YOU MIGHT ALSO LIKE... NCLEX-RN Exam | Mometrix Comprehensive Guide21 sets Mometrix $23.99 STUDY GUIDE Maternal Newborn Final40 Terms atomant17 Maternal Newborn Final40 Terms kshikita UW: 4 Maternal and Newborn Health85 Terms ogbaami Terms in this set (57) A nurse is teaching a client who has starting a new prescription for an oral contraceptive. Which of the following medications are contraindicated for this client? Anticonvulsants A client in her third trimester of pregnancy is attending childbirth classes. The nurse concludes that the client understood her teaching about Kegel exercises when she states, "These exercises will help my pelvic muscles stretch when I give birth." A client who is at 11 weeks of gestation comes to the clinic stating that she has had slight occasional vaginal bleeding over the past 2 weeks. The provider determines that the fetus has died and that the placenta, fetus, and tissues remain in the uterus. How should the nurse document these findings? Missed miscarriage A nurse is caring for a newborn who is small for gestational age (SGA). Which of the following priority interventions should the nurse add to this newborn's plan of care? Monitor blood glucose levels. A nurse on the postpartum unit is caring for a client who experienced abruptio placentae. The nurse observes petechiae and bleeding around the IV access site. The nurse recognizes this client is at risk for which of the following postpartum complications? Disseminated intravascular coagulation A nurse in labor and delivery is caring for a client on an external electronic fetal monitor. The nurse observes the fetal heart rate begins to decelerate after the contraction has started and the lowest point of the deceleration occurs after the peak of the contraction. Which of the following is the priority action by the nurse? Place the client in the lateral position. Only $1/month A nurse is caring for a newborn who has respiratory distress syndrome and is experiencing respiratory acidosis. The nurse is aware that respiratory acidosis is caused by which of the following? Inadequate chest expansion A client delivered a 34-week, 1,550-g newborn who has nasal flaring, intercostal retractions, expiratory grunting, and mild cyanosis. The nurse should place the newborn in an incubator that will create a neutral thermal environment because his temperature control mechanism is immature. A nurse is assessing a newborn 1 hr after delivery. Which of the following respiratory rates is within the expected reference range for a newborn? 48/ min A nurse is caring for a client who is in premature labor and is receiving terbutaline (Brethine) as a tocolytic agent. The nurse should know to closely monitor the client for which of the following side effects of terbutaline that indicates the development of a potentially life-threatening complication? Palpitations he nurse is caring for a client in the prenatal clinic who is in her seventh week of pregnancy. The client reports urinary frequency and asks if this will continue throughout the pregnancy. Which of the following is the correct response by the nurse? "It should only last until about your 12th week, but it will return near the end of the pregnancy." A nurse is caring for a client who is postpartum. The nurse should recognize which of the following statements by the client as a possible indication of inhibition of parental attachment? "I just wish he had more hair. I'm going to have to keep a hat on his head till he grows some." A nurse is assessing a 7-month-old infant. Which of the following would indicate the need for further evaluation? Babbles one-syllable sounds A nurse is completing a newborn gestational age assessment. Which of these findings is recorded as part of this assessment? plantar creases cover 2.3 of sole A nurse is performing a vaginal examination of a client who is in active labor, with the fetus in vertical lie. When the nurse determines that the fetal head is at station 0 it means that the lowermost portion of the head is at the level of the ischial spines. A nurse is caring for a client who is in active labor at 39 weeks of gestation. The nurse locates the fetal heart tones above the client's umbilicus at midline. The nurse should suspect that the fetus is in which of the following positions? Frank breech Instructing a client about her nutritional needs during pregnancy, the nurse tells her that she will have an increased need for iron A nurse is caring for a client who is at 37 weeks of gestation and has placenta previa. The client tells the nurse that the provider said he cannot examine her internally and the client asks the nurse why. The nurse should explain that this is primarily because an internal examination could cause profound bleeding. A nurse in a hospital is caring for a client who is at 38 weeks of gestation and has a large amount of painless, bright red vaginal bleeding. The client is placed on a fetal monitor that shows a fetal heart rate that is 138/min and regular, and no obvious uterine contractions are noted. The client's vital signs are obtained: blood pressure 98/52 mm Hg, pulse 118/min, respirations 24/min, and temperature 36.4° C (97.6° F). The nurse should give first priority to implementing which of the following actions? Start an IV of normal saline. A nurse instructs a female client on how to check basal temperature in order to determine if the client is ovulating. The nurse tells the client to check her temperature every morning before arising. The nurse is caring for a client who is 5 hr postpartum following a vaginal birth of a newborn weighing 9 lb 6 oz. (4252 g). The nurse recognizes this client is at risk for which of the following postpartum complications? Uterine atony 205. A nurse is caring for a client who is postpartum. Following delivery of the placenta, the nurse examines the umbilical cord. Which of the following vessels should the nurse expect to observe in the newborn's umbilical cord? Two arteries and one vein A nurse is assessing an infant. Which of the following is a clinical manifestation of a large patent ductus arteriosus? Machinery-like murmur A nurse is providing teaching to a client who is pregnant and has phenylketonuria (PKU). Which of the following foods should Peanut butter A nurse in a prenatal clinic is assessing a client who is suspected of having a hydatidiform mole. Which of the following findings should the nurse expect to observe in this client? Excessive uterine enlargement A nurse is caring for a client who is in labor and has an external fetal monitor. The nurse notes late decelerations and interprets them as indicating which of the following? Potential for fetal distress A nurse is caring for a client who has a possible ectopic pregnancy at 8 weeks of gestation. Which of the following manifestations should the nurse expect to identify as consistent with the diagnosis? Unilateral crampy abdominal pain A nurse is caring for a client who is in her first trimester of pregnancy and asks the nurse if she can still ride her bike for exercise. The nurse's response should be "You may continue, but be careful not to go to the point of exhaustion." A nurse is assisting a client with breastfeeding her newborn. The nurse should explain that which of the following reflexes will initiate sucking? Rooting A nurse is assessing a client who is 3 days postpartum and is breastfeeding. Her fundus is three fingerbreadths below the umbilicus, and her lochia rubra is moderate. Her breasts feel hard and warm. The nurse should identify that the client has expected findings at this time. A nurse is caring for a client who just delivered a newborn. Following the delivery, which nursing action should be done first to care for the newborn? Clear the respiratory tract. A nurse is caring for a client who is at 28 weeks of gestation and has received terbutaline (Brethine). Which of the following findings should the nurse expect? Weakened uterine contractions (toco med; slows down labor) A nurse is caring for a newborn and calculating the Apgar score. At one-minute after delivery, the following is noted: heart rate of 110; slow, weak cry; some flexion of extremities; grimace in response to suctioning of the nares; body pink in color with blue extremities. Calculate the Apgar score. (Fill in the blank.) 6 A nurse is caring for a newborn client whose mother has the 3-day measles at the time of delivery. Which of the following statements made by the nurse indicates that she understands the correct rational for placing the newborn in isolation? "The newborn may be actively shedding the virus." A nurse is planning care for an infant that has been diagnosed with phenylketonuria (PKU). Which of the following is an appropriate action for the nurse to take? Initiate a controlled diet eliminating protein. A nursery nurse is admitting a term newborn following a cesarean birth. The nurse observes that the infant's skin is slightly yellow. This finding indicates the newborn is experiencing a complication related to which of the following? Maternal/newborn blood group incompatibility A nurse is caring for a client who is in the first stage of labor. The client has been experiencing contractions that have a duration of 60 seconds and a frequency of 3 min. Upon examination, the nurse notes a glistening white cord in the vagina. Which of the following actions should the nurse perform first? Place the client in Trendelenburg position. (b/c there is no circulation passing through the cord, this will help put no pressure on their cord aka more air) A nurse is caring for a client who is 6 hr postpartum. The client is Rh-negative and her newborn is Rh-positive. The client asks why an indirect Coombs test was ordered by the provider. Which of the following is an appropriate response? "It detects Rh positive antibodies in the mother's blood." A nurse is reinforcing teaching about newborn care with a postpartum client. Which of the following statements by the client indicates a need for further teaching? "Baby powder will help prevent a diaper rash." A provider prescribes methylergonovine (Methergine) IM for a client who had a vaginal delivery earlier that day. The nurse should explain to the client that this medication will help prevent postpartum hemorrhage. A nurse in a prenatal clinic is caring for a client at 12 weeks gestation. The client asks about the cause of her heartburn. Which of the following is the appropriate response by the nurse? Increased progesterone production causes decreased motility of smooth muscle. A nurse in a prenatal clinic is caring for a woman at 39 weeks gestation who asks about the signs that precede the onset of labor. Which of the following is an appropriate response by the nurse? a surge of energy A client who is 1 day postpartum is having a sitz bath. To determine the client's tolerance of the procedure, which of the following assessments should the nurse perform? pusle rate A client at 20 weeks of gestation tells the nurse at the prenatal clinic that she is concerned that exercising might pose risks to her pregnancy. Which of the following is an appropriate nursing response? "Moderate exercise can help improve your circulation." A nurse is teaching a parent of an infant about bottle-feeding. Which of the following statements by the parent indicates a need for further teaching? "Each feeding should last about 15 minutes." A nurse is caring a client who is 1 day postpartum and is attempting to breastfeed. Which of the following findings indicate mastitis? Red and painful area in one breast A client who is at 22 weeks of gestation has been unable to control her gestational diabetes with diet and exercise. The nurse should explain to the client that it is likely that the provider will prescribe which of the following medications to control her blood glucose levels? Regular insulin (Humulin R) A 14-year-old client attends a class about contraception at a community clinic. After the class, she asks the nurse which method is best for her to use. Which of the following is an appropriate nursing response? "Before I can help you with that question, I need to know more about your sexual activity." A nurse is reinforcing teaching to a client who is at 30 weeks of gestation and scheduled for a nonstress test (NST). Which of the following statements by the client indicates a need for further teaching? "I will have to lie on my back during the test." A nurse is instructing a male client about a semen analysis to be done for suspected infertility. Which of the following should be included in the teaching? Abstain from ejaculation for at least 2 to 5 days prior to the test. A nurse is preparing to administer oxygen via hood therapy to an infant who was born at 30 weeks gestation. Which of the following is the highest safe level of oxygen that can be administered to a premature infant? 70 percent A nurse in the nursery is caring for a newborn. The grandmother of the newborn asks if she can take the newborn to the mother's room. Which of the following is an appropriate response by the nurse? "Have the mother call and I will take the baby to the room." Two hours after a spontaneous vaginal delivery, a client has saturated two perineal pads with blood in a 30- min period. Which of the following actions is the priority for the nurse to take at this time? Check the consistency of the client's uterine fundus. A nurse is caring for a client in the first trimester of pregnancy. After reviewing the client's blood work, the nurse notices she does not have immunity to rubella. The nurse instructs the client that the appropriate time for immunization is shortly after giving birth. A nurse is caring for a client who is beginning to breastfeed her newborn after delivery. The new mother states, "I don't want to take anything for pain because I am breastfeeding." Which of the following is an appropriate response by the nurse? "We can time your pain medication so that you have an hour or two before the next feeding." A nurse is talking with a client who is at 23 weeks of gestation and will return to the facility in 2 days for an amniocentesis. Which of the following instructions should the nurse give the client? Empty her bladder immediately prior to the procedure. A nurse is caring for a client who is 2 days postpartum, is breastfeeding, and reports nipple soreness. Which of the following measures should the nurse suggest to help lessen discomfort during breastfeeding? (Select all that apply.) A. Apply breast milk to her nipples before each feeding. B. Alternate breasts at the beginning of each feeding. D. Start breastfeeding with the nipple that is less sore. E. Change the infant's position on the nipples. Terms in this set (25) Dry the skin A nurse is caring for a newborn immediately following birth. After assuring a patent airway, what is the priority nursing action? - Hepatitis B immunization - Vitamin K injection - antibiotic ointment to both eyes A nurse is assisting with the care of a newborn immediately following birth. Which of the following medications should the nurse anticipate administering? Assess the uterus for position and consistency and message the woman's fundus On examination a woman who have birth 5 hours ago, the nurse finds that the woman has completely saturated a perineal pad within within 15 minutes. The nurse's first action is to: Unilateral, abdominal pain A nurse is caring for a client who has suspected ectopic pregnancy at 8 weeks of gestation. Which of the following manifestations should the nurse expect to identify as consistent with the diagnosis? Fundus three finger breaths above the umbilicus A nurse is assessing a client who is 8 hr postpartum and multiparous. Which of the following findings should alert the nurse to ( i cant read the question)... need to urinate? Hemorrhage is the major concern A 32-year-old primigravida is admitted with a diagnosis of ectopic pregnancy. Nursing care is based on the knowledge that: Excessive uterine enlargement A nurse in a prenatal clinic is caring for a client who is suspected of having a hydatidiform mole. Which of the following findings should the nurse expect to observe in this client? "Your baby should wet 6 to 8 diapers per day" A nu A nurse is completing discharge instructions for a new mother and her 2-day-old newborn. The mother asks, "How will I know if my baby gets enough breast milk?" Which of the following responses should the nurse make? - History of migraines - Twin gestations - First pregnancy A nurse in a clinic is assessing a client who is at 8 weeks of gestation and has hyperemesis gravidarum. Which of the following findings should the nurse expect? - Apply breast milk to the nipples before each feeding - Change the infant's position protect the nipple - Start breastfeeding with the nipple that is less sore A nurse is caring for a client who is 2 days postpartum, is breastfeeding, and reports nipple soreness. Which of the following measures should the nurse suggest to reduce discomfort during breastfeeding? (Select all that apply) A caput succedaneum occurs due to compression of blood vessels. (I cannot find the question) 6 A nurse is caring for a newborn and calculating the Apgar score. At 1 min after delivery, the following findings are noted: heart rate if 110/min; slow, weak cry; some flexion of extremities; grimace in response to suctioning of the nares; body pink in color with blue extremeities. Calculate the newborn's Apgar score and fill in the blank: points Fundus firm, at the level of the umbilicus A nurse is caring for a client who is 12 hr postpartum following a vaginal delivery. Which of the following findings should the nurse expect? Feeding an infant can feel a little intimidating at first, but I'll stay and help you A nurse is caring for a client who is 6 hours postpartum and asks the nurse to feed her newborn. Which of the following responses should the nurse provide? Cullen's sign A nurse in the emergency department is caring for a client who comes to the emergency department reporting severe abdominal pain in the left lower quadrant. The provider suspects a ruptured ectopic pregnancy. Which of the following signs indicates to the nurse that the client has blood in the peritoneum? - Clean the perineal area from front to back. - Perform hand hygiene before and after voiding. - Wash the perineal area using a squeeze bottle of warm water after each voiding. - Blot the perineal area dry after cleansing. A nurse is reinforcing teaching about reducing perineal infection with a client following a vaginal delivery. Which of the following should the nurse include in the teaching? (Select all that apply) "I should remove extra blankets from my baby's crib." A nurse is teaching about crib safety with the parent of a newborn. Which of the following statements by the client indicates understanding of the teaching? Red and painful area in one breast A nurse is caring for a client who is 3 days postpartum and is attempting to breastfeed. Which of the following indicate mastitis? - Observe the lochia during palpation of fundus. - Determine whether the fundus is midline. - Massage uterus if boggy. - Document fundal height. A nurse is caring for a client who had a vaginal delivery 2 hr ago. Which of the following actions should the nurse anticipate in the care of this client? - Provide a dark, quiet environment. - Ensure that calcium gluconate is readily available. - Administer magnesium sulfate IV. A nurse is admitting a client who is at 37 weeks of gestation and has severe gestational hypertension. Which of the following actions should the nurse expect to implement? (Select all that apply) 48/min A nurse is assessing a newborn 1 hr after birth. Which of the following respiratory rates is within the expected reference range for a newborn? Necrotizing enterocolitis A nurse is caring for a preterm newborn who has a nasogastric tube and who recently began intermittent gavage feedings of formula. The nurse notes increased abdominal distention, lethargy, bloody stools, and increasing gastric residuals before feedings. The nurse should suspect which of the following? Monitor blood glucose levels. A nurse is planning care for a newborn who is small for gestational age (SGA). Which of the following is the priority intervention the nurse should include in the newborn's plan of care? Your baby should wet 6 to 8 diapers per day A nurse is completing discharge instructions for a new mother and her 2-day-old newborn. The mother asks, "How will I know if my baby gets enough breast milk?" Which of the following responses should the nurse make? Rooting A nurse is assisting a client with breastfeeding. The nurse explains that which of the following reflexes will promote the newborn to latch? A nurse is assessing a client is at 35 weeks of gestation and has mild gestational hypertension. Which of the following findings should the nurse identify as the priortiy? 480 mL urine output in 24 hr A nurse is reviewing the medical record of a client who is at 33 weeks gestation and has placenta previa and bleeding. Which of the following prescriptions should the nurse clarify with the provider? Perform a vaginal examination. A nurse is assessing a client who is at 35 weeks of gestation and is receiving magnesium sulfate via continuous IV infusion for severe pre-eclampsia. Which of the following findings should the nurse report to the provider? Urinary output 20 mL/Hr A nurse is assessing a client who is at 12 weeks of gestation and has a hydatidiform mole. Which of the following findings should the nurse expect? Dark brown vaginal discharge A nurse is caring for a client who is at 37 weeks of gestation and is undergoing a nonstress test. The FHR is 130/min without accelerations for the past 10 min. Which of the following actions should the nurse take? Use vibroacoustic stimulation on the client's abdomen for 3 seconds. A nurse is caring for a client who is at 26 weeks of gestation and reports constipation. Which of the following responses by the nurse is appropriate? "You should walk for at least 30 minutes every day." A nurse is teaching a client who is at 12 weeks of gestation about manifestations of potential complications that she should report to her provider. Which of the following information should the nurse include in the teaching? Swelling of the face A nurse is caring for a client who is at 35 weeks of gestation and has severe pre-eclampsia. Which of the following assessments provides the most accurate information regarding the client's fluid and electrolyte status? Daily weight A nurse is providing teaching to a client who is at 8 weeks of gestation about manifestations to report to the provider during pregnancy. Which of the following information should the nurse include in the teaching? Blurred or double vision A nurse is caring for a client who believes she may be pregnant. Which of the following findings should the nurse identify as a positive sign of pregnancy? Palpable fetal movement A nurse is teaching a client who is at 8 weeks of gestation and has a uterine fibroid about potential effects of the fibroid during pregnancy. Which of the following information should the nurse include in the teaching? The fibroid can increase the risk for postpartum hemorrhage A nurse is caring for a client whose last menstrual period (LMP) began July 8. Using Nagele's rule, the nurse should identify the client's estimated birth (EDB) as which of the following? April 15 A nurse at as prenatal clinic is caring for a client who suspects she may be pregnant and asks the nurse how the provider will confirm her pregnancy. The nurse should inform the client that which of the following laboratory tests will be used to confirm her pregnancy? A urine test for the presence of human chorionic gonadotropin (HCG) A nurse is caring for a client who is at 32 weeks of gestation and is experiencing preterm labor. Which of the following medications should the nurse plan to administer? Betamethasone A nurse is caring for a client who is in the latent phase of labor and is experiencing low back pain. Which of the following actions should the nurse take? Apply pressure to the client's sacral area during contractions. A nurse is teaching a client who is at 12 weeks of gestation and has human immunodeficiency virus (HIV). Which of the following statements should the nurse include in the teaching? "You should continue to take zidovudine throughout the pregnancy." A nurse is admitting a client who is in labor and experiencing moderate bright red vaginal bleeding. Which of the following actions should the nurse take? Obtain blood samples for baseline laboratory values A nurse is teaching a client who is at 10 weeks of gestation about an abdominal ultrasound in the first trimester. Which of the following information should the nurse include in the teaching? "You will need to have a full bladder during the ultrasound." A nurse is caring for a client who is in the latent phase of labor and is receiving oxytocin via continuous IV infusion. The nurse notes that the client is having contractions every 2 min which last 100 to 110 seconds and that the FHR is reassuring. Which of the following actions should the nurse take? Decrease the dose of oxytocin by half A nurse is caring for a client who is at 38 weeks of gestation and reports no fetal movement for 24 hr. Which of the following actions should the nurse take? Auscultate for a fetal heart rate A nurse is teaching a client who has pre-eclampsia and is to receive magnesium sulfate via continuous IV infusion about expected adverse affects. Which of the following adverse effects should the nurse include in the teaching? Feeling of warmth A nurse is reviewing laboratory results for a client who is at 37 weeks of gestation. The nurse notes that the client is rubella non-immune, positive for group A beta-hemolytic streptococci, and has a blood type of O negative. Which of the following actions should the nurse take? Instruct the client to obtain a rubella immunization after delivery A nurse is reviewing the medical record of a client who is at 39 weeks of gestation and has polyhydramnios. Which of the following findings should the nurse expect? Fetal gastrointestinal anomaly A nurse is teaching a client who is at 30 weeks of gestation about warning signs of complications that she should report to her provider. Which of the following findings should the nurse include in the teaching? Vaginal bleeding A nurse is assessing a client who is at 34 weeks of gestation and has mild placental abruption. Which of the following findings should the nurse expect? Dark red vaginal bleeding A nurse is caring for a client who is at 39 weeks of gestation and is in the active phase of labor. The nurse observes late decelerations in the FHR. Which of the following findings should the nurse identify as the cause of late decels? Uteroplacental insufficiency? A nurse is teaching a client who is at 13 weeks of gestation about the treatment of incompetent cervix and cervical cerclage. Which of the following statements by the client indicates as understanding of the teaching? "I should go to the hospital if I think I may be in labor." A nurse is caring for a client who has oligohydramnios. Which of the following fetal anomalies should the nurse expect? Renal agenesis A nurse is assessing a client who is at 37 weeks of gestation and has a suspected pelvic fracture due to blunt abdominal trauma. Which of the following findings should the nurse expect? Uterine contractions A nurse is caring for a client who is in active labor and has meconium staining of the amniotic fluid. The nurse notes a reassuring fetal heart rate (FHR) tracing form the external fetal monitor. Which of the following actions should the nurse take? A nurse is assessing a client on the first postpartum day. Findings include fundus firm and one fingerbreadth above to the the right of umbilicus, moderate lochia rubra with small clots, temp 37.3 (99.2 F), pulse 52/min. Which of the following actions should the nurse take? A. report vital signs to the provider B. massage the fundus C. ask client when she last voided D. administer oxytocin agent C. ask the client when she last voided Fundus is easily displaced when the bladder is full. Should be found firm at midline. A nurse is preparing to administer naloxone to a newborn. which of the following conditions can require administration of this medication? A. IV narcotics administered to the mother during labor B. maternal drug use C. hyaline membrane disease D. meconium aspiration A. IV narcotics administered to the mother during labor Should administer naloxone to reverse respiratory depression due to acute narcotic toxicity, which can result from IV narcotics administration during labor A nurse is discussing anesthesia with a client who is receiving oxytocin for induction of labor. Which of the following statements should the nurse make? A. an epidural given too early in labor can cause maternal hypertension B. an epidural given too early during labor will not be effective in active labor C. an epidural given too early can cause fetal depression D. an epidural given too early can prolong labor D. an epidural given too early can prolong labor Clients who receive anesthesia before the active phase of labor usually find the progression to be slow. The medication depresses the CNS, so it takes longer for the cervix to dilate and efface A nurse is caring for a client who is pregnant and reports nausea and vomiting. Which of the following instructions should the nurse provide the client? A. you should eat some crackers before rising from bed in the morning B. you should eat foods served at warm temperature C. you should sip whole milk withe breakfast D. you should brush your teeth immediately after meals A. you should eat some crackers before rising from bed in the morning morning sickness is caused by the buildup of hcg in the mother's system. dry foods eaten before rising in the morning tend to reduce the risk of nausea in clients who are pregnant. A nurse is planning care for a client who is pregnant and is Rh-. In which of the following situations should the nurse administer Rhogam? A. while the client is in labor B. following an episode of influenza during pregnancy C. prior to a blood transfusion D. at 28 weeks of gestation D. at 28 weeks of gestation Rhogam consists of passive antibodies against the Rh factor, which will destroy any fetal RBCs in the maternal circulation and block maternal antibody production A nurse is caring for a newborn whose mother received magnesium sulfate to treat preterm labor. Which of the following clinical manifestations in the newborn indicates toxicity due to the magnesium sulfate therapy? A. respiratory depression B. hypothermia C. hypoglycemia D. jaundice A. respiratory depression Magnesium sulfate can cause resp. and neuromuscular depression in the newborn. The nurse should monitor the newborn for clinical manifestations of resp. depression Only $1/month A nurse is caring for a newborn who was born to a client who has a narcotic use disorder. Which of the following nursing actions should the nurse identify as a contraindication for the care of the newborn? A. promoting maternal-newborn bonding B. tight swaddling of the newborn C. small frequent feedings D. A D. frequent stimulation This newborn needs a quiet, calm environment with minimal stimulation to promote rest and reduce stress. A stimulating environment can trigger irritability and hyperactive behaviors A nurse is caring for a client who is in labor. A vaginal examination reveals the following info: 2 cm, 50%, +1 right occiput anterior. Based on this info, which of the following fetal positions should the nurse document in the medical record? A. transverse B. breech C. vertex D. mentum C. vertex ROA describes the relationship of the presenting part of the fetus to the client's pelvis. In this case, the occipital bone is the presenting part and is located anteriorly in the client's right side. Based on the presentation of the fetus, the position is vertex A nurse is caring for a client who desires an IUD for contraception. Which of the following findings is a contraindication for the use of this device? A. htn B. menorrhagia C. history of multiple gestations D. history of thromboembolic disease C. menorrhagia contraindication for women who have menorrhagia, severe dysmenorrhea, history of ectopic pregnancy A nurse is caring for a client who is at 39 weeks of gestation and is in active labor. Which of the following actions should the nurse include in the plan of care? A. keep four side rails up while the client is in bed B. monitor FHR every hour C. insert indwelling urinary catheter D. check the cervix prior to analgesic administration D. check the cervix prior to analgesic administration The nurse must know how many cm the cervix is dilated prior to analgesic administration during active labor. If administred too close to the time of delivery, the analgesic could cause resp. depression in the newborn A nurse is caring for a client who has trichomoniasis and a prescription for metronidazole. Which of the following should the nurse provide to the cilent about the treatment plan? A. your partner needs to be cultured and be treated with metronidazole only if his cultures are positive B. you and your partner need to take the medication and use a condom during intercourse until cultures are negative C. if both you and your partner are treated simultaneously, you may continue to engage in sexual intercourse D. only you will need to take the metronidazole, but you should not have intercourse until your culture is negative B. you and your partner need to take the medication and use a condom during intercourse until cultures are negative Both men and women can be infected with trichomoniasis. Clinical findings include yellowish to greenish, frothy, mucopurulent, copious discharge with an unpleasant odor, as well as itching, burning, or redness of the vulva and vagina. Easily treated with metronidazole. A nurse is caring for four newborns. Which of the following newborns is at greatest risk for hypoglycemia? A. a newborn who is large for gestational age B. a newborn who has an Rh incompatibility C. a newborn who has pathologic jaundice D. a newborn who has fetal alcohol syndrome A. a newborn who is large for gestational age LGA newborns are those whose weight is at or above the 90th percentile. One of the most common etiologies of LGA is a mother who is diabetic. LGA newborns, especially those born to mothers who have diabetes, are at increased risk for hypoglycemia. Other newborns at risk for hypoglycemia are SGA, premature newborns, and newborns who have perinatal hypoxia. A nurse is caring for a client who is 2 hr postpartum. The nurse notes the client's perineal pad has a large amount of lochia rubra with several clots. Which of the following actions should the nurse take first? A. check for a full bladder B. massage the fundus C. measure vital signs D. administer carboprost IM B. massage the fundus The primary cause of early postpartum bleeding is uterine atony manifested by a relaxed, boggy uterus. The greatest risk for the client is hemorrhage. The nurse should massage the client's fundus first. A nurse is caring for a client whose membranes have ruptured and is in active labor. The fetal monitor tracing reveals late decelerations. Which of the following actions should the nurse take first? A. turn the client on her left side B. palpate the client's uterus C. administer o2 to the client D. increase the client's IV fluids A. turn the client onto her left side Late decelerations indicate that the client is experiencing uteroplacental insufficiency. The client might be experiencing pressure on the inferior vena cava, which decreases teh o2 to the placenta and thus the fetus. Turning the client on her left side will relieve the pressure and facilitate better flood flow to the placenta, therby increasing the fetal o2 supply A nurse is planning care for a client who has a prescription for oxytocin. which of the following is a contraindication for the use of this medication? A. prolonged rupture of membranes at 38 weeks of gestation B. intrauterine growth restriction C. postterm pregnancy D. active genital herpes D. active genital herpes The newborn can acquire genital herpes as they pass through the birth canal, so oxytocin is contraindicated. A c-section is recommended if genital herpes is active A nurse is caring for a newborn who has neonatal abstinence syndrome. Which of the following clinical findings should the nurse expect? A. extended periods of sleep B. poor muscle tone C. respiratory rate 50/min D. exaggerated reflexes D. exaggerated reflexes A newborn who has neonatal abstinence syndrome usually exhibits clinical findings of hyperactivity within the CNS. Exaggerated reflexes are indicative of CNS irritability. Newborn would also have increased muscle tone, respiratory distress with rate greater than 60/min, A nurse receives report on a client who is in labor and is experiencing contractions 4 min apart. Which of the following patterns should the nurse expect on the fetal monitoring tracing? A. contractions that last for 60 seconds each with a 4-min rest between contractions B. contractions that last for 60 seconds each with a 3-min rest between contractions C. a contraction that lasts 4 min followed by a period of relaxation D. contractions that last 45 seconds each with a 3-min rest between contractions B. contractions that last for 60 seconds each with a 3-min rest between contractions A nurse is caring for a client who has clinical manifestations of an ectopic pregnancy. Which of the following findings is a risk factor for an ectopic pregnancy? A. anemia B. frequent urinary tract infections C. previous c-section D. pelvic inflammatory disease (PID) D. PID The most common site in ectopic pregnancies is within a fallopian tube, but can also occur in the ovary or the abdomen. Most cases are a result of scarring caused by a previous tubal infection or tubal surgery. A nurse is caring for a client who is at 8 weeks of gestation with twins and primigravida. The client states that even though she and her husband planned this pregnancy, she is experiencing many ambivalent feelings about it. Which of the following responses should the nurse make? A. have you told your husband about these feeligns B. these feelings are quite normal at the beginning of pregnancy C. perhaps you should see a counselor to discuss these feelings D. i am quite concerned about these feelings. could you explain more? B. these feelings are quite normal at the beginning of pregnancy The client needs reassurance that these feelings are normal and there is no reason for concern. A nurse is assessing a newborn who is 12 hr old and noes mild jaundice of the face and trunk. Which of the following actions should the nurse take? A. administer phytonadione IM B. obtain a stat prescription for a bilirubin level C. obtain a bagged urine specimen D. perform a gestational age assessment B. obtain a stat prescription for a bilirubin level Jaundice in the first 24 hr of life is pathologic. The nurse should notify the provider and obtain a stat prescription for a bilirubin level A nurse is teaching a client who has active genital herpes simplex virus, type 2. Which of the following statements should the nurse include in the teaching? A. you will have a c-section prior to the onset of labor B. your baby will receive erythromycin eye ointment after birth to treat the infection C. you should take oral metronidazole for 7 days prior to 37 weeks of gestation D. you should schedule a c-section after your water breaks A. you will have a c-section birth prior to the onset of labor Whenever possible, the c-section birth should be scheduled prior to the onset of labor or rupture of membranes to reduce the risk of neonatal transmission of herpes A nurse is caring for a client who has prescription for naloxone. Which of the following is the intended action of the medication in relation to the CNS? A. accentuate effects of narcotics on the CNS B. depress activity of the CNS C. block effects of narcotics on the CNS D. stimulate activity of the CNS C. blocks effects of narcotics on the CNS By blocking the effects of narcotics on the CNS, naloxone prevents CNS and respiratory depression in the newborn following delivery A nurse in a prenatal clinic is caring for a client who is within the recommended guideline for weight. The client asks the nurse how much weight is safe for her to gain during her pregnancy. Which of the following responses should the nurse make? A. your provider can discuss an appropriate amount of weight gain with you B. a weight gain of about 14 pounds each trimester is suggested C. if you eat nutritious food when you feel hungry, the amount of weight gain is insignificant D. a weight gain of about 25-35 pounds is good D. a weight gain of about 25-35 pounds is good A weight gain of 25-35 lbs is associated with good fetal outcome. A gain of 4 lb in the first trimester and 12 lb for the second and third trimester is recommended A nurse is caring for a client who is in labor and has an epidural for pain relief. Which of the following is a complication from the epidural block? A. n/v B. tachycardia C. hypotension D. respiratory depression C. hypotension maternal hypotension is an adverse effect of epidural anesthesia. The nurse should administer an IV fluid bolus prior to the placement of epidural anesthesia in order to decrease the likelihood of this complication A nurse is providing discharge teaching to a client following the removal of a hydatidiform mole. Which of the following statements should the nurse include in the teaching? A. do not become pregnant for at least 1 year B. seek genetic counseling for yourself and your partner prior to getting pregnant again C. you should have an hCG level drawn in 6 weeks D. have your blood pressure checked weekly for the next month A. do not become pregnant for at least 1 year Hydatidiform moles are uncontrolled growths in the uterus arising from placental or fetal tissue in early pregnancy. There is increased incidence of choriocarcionma associated with molar pregnancies. Pregnancy must be avoided for 1 year so the client can be closely monitored for manifestations of the condition. A baseline hCG level should be obtained following evacuation of the mole and then weekly until levels are normal for 3 consecutive weeks. Additional hCG levels should be obtained every 4 weeks for the next 6-12 months A nurse is planning care for a client who is at 35 weeks of gestation. Which of the following laboratory tests should the nurse obtain? A. rubella titer B. blood type C. group B strep B-hemolytic D. 1-hour glucose tolerance test C. group B streptococcus B-hemolytic Should obtain a vaginal/anal GBS culture at 35-37 weeks of gestation to screen for infection. prophylactic antibiotics should be given during labor to the client who is positive for GBS. Rubella titer at first prenatal visit Blood type and Rh factor at first prenatal visit to determine if Rhogam is given at 28 weeks 1 hr GTT at 24-28 weeks to screen for gestational diabetes A nurse is caring for a client who is at 34 weeks of gestation and has a prescription for terbutaline for preterm labor. Which of the following statements by the client is the priority? A. my ankles are swollen at the end of the day B. i can feel the baby kicking my ribs, and it is very uncomfortable C. i'm growing more and more worried every day D. my heart feels as if it is racing D. my heart feels as if it is racing The primary action of terbutaline is to cause bronchodilation and relax smooth muscle. However, an adverse effect is tachycardia. If the pulse is greater than 130/min, the terbutaline needs to be held until the provider is notified. A nurse is planning care for a newborn who requires phototherapy for hyperbilirubinemia. Which of the following actions should the nurse include in the plan of care? A. swaddle the newborn in a receiving blanket during the treatment B. maintain NPO status until the newborn's bilirubin is within the expected reference range C. ensure the newborn's eyes are closed before applying the eye shield D. apply lotion to the newborn's skin twice per day C. ensure the newborn's eyes are closed before applying the eye shield Overexposure to the lights during treatment can cause damage to the newborn's corneas. Therefore, the nurse should gently close the newborn's eyes prior to applying the eye shield. A nurse administers betamethasone to a client who is at 33 weeks of gestation to stimulate fetal lung maturity. When planning care for the newborn, which of the following conditions should the nurse identify as an adverse effect of this medication? A. hyperthermia B. decreased blood glucose C. rapid pulse rate D. irritability B. decreased blood glucose Betametasone causes hyperglycemia, which predisposes the newborn to hypoglycemia in the first hours after delivery. It is important to assess the newborn's blood glucose level within the first hour following birth and frequently thereafter until blood glucose levels are stable. Does not have an affect on the newborn's vital signs A nurse is caring for a client who is at 16 weeks of gestation and has severe iron-deficiency anemia. The provider prescribes an injection of iron dextran IM. Which of the following methods should the nurse use to administer the medication? A. 20-gauge needle, administer using z-track method B. 22-gauge needle, administer deep into the thigh C. use a 25-gauge needle, administer into deltoid muscle D. 18-gauge needle, administer into rectus femoris muscle A. use a 20-gauge needle, and administer the med using z-track method The nurse should administer iron using the z-track method to prevent staining of the tissue. 20- gauge needle is the correct size. Use ventrogluteal muscle A nurse in an antepartum clinic answers a phone call from a client who is at 37 weeks of gestation and reports, "I became very dizzy while lying in bed this morning, but the feeling went away when I turned on my side." Which of the following actions should the nurse take? A. instruct the client about vena cava syndrome and measures to prevent it B. arrange for the client to come to the clinic for assessment C. check the client's chart for gestational diabetes mellitus D. schedule a NST A. instruct the client about vena cava syndrome and measures to prevent it A nurse is teaching a client about a nonstress test. Which of the following statements by the client indicates an understanding of the teaching? A. i know not to eat anything after midnight B. i will have medication given to me to cause contractions C. i should press the button on the handheld marker when my baby moves D. i will stimulate my breast to cause contractions C. i should press the button on the handheld marker when my baby moves The purpose of the test is to assess fetal well-being. The client should press the button on the handheld marker when she feels fetal movement A nurse is caring for a client who is at 36 weeks of gestation and has pre-eclampsia. Which of the following findings should the nurse identify as the priority? A. 1+ proteinuria B. blood pressure 140/98 mm Hg C. nonreactive nonstress test D. fundal height 33 cm C. nonreactive nonstress test A nonstress test measures FHR accelerations with normal movement. A fetal accelration is a positive sign present when the FHR increases 15/min and lasts 15 seconds. In a nonreactive nonstress test, there are no accelerations. Absence of FHR accelerations suggests that the fetus might be going into distress. A nurse is caring for a client who is in labor. The client questions the application of an internal fetal scalp electrode. Which of the following responses should the nurse make? A. don't worry, your baby is fine B. you will need to ask your provider C. your provider feels it would be best D. we need to observe your baby more closely D. we need to observe your baby more closely The client has asked an information-seeking question. This therapeutic response provides info to the client in an honest, nonthreatening manner. A nurse is assessing a client who is receiving magnesium sulfate as treatment for pre-eclampsia. Which of the following clinical findings is the nurse's priority? A. resp. 16/min B. urinary output 40 mL in 2 hr C. reflexes +2 D. fetal heart rate 158/min urinary output 40 ml in 2 hr Urinary output is critical to the excretion of magnesium from the body. The nurse should discontinue the magnesium sulfate if the hourly output is less than 30 ml/hr A nurse is speaking with an expectant father who says that he feel resentful of the added attention others are giving to his wife since the pregnancy was announced several weeks ago. Whihc of the following responses should the nurse make? A. has your wife sensed your anger toward her and the baby B. these feelings are common to expectant fathers in early pregnancy C. i'm sure that it's really hard to accept this when it's your baby, too D. it would be wise for you to peak to a therapist about these feelings B. these feelings are common to expectant fathers early in pregnancy A nurse is caring for a client who is receiving oxytocin for induction of labor. Which of the following actions should the nurse take? A. perform continuous FHR monitoring B. measure maternal temp every hour C. evaluate maternal contraction pattern every hour D. check bp every 5 min A. perform continuous FHR monitoring When oxytocin is administered to an antepartum client, the FHR must be used to continuously monitor the FHR and maternal contractions A nurse is discussing diaphragm use with a client. Which of the following statements by the client indicates an understanding of the teaching? A. i should clean my diaphragm with alcohol each time i use it B. i should leave the diaphragm in place 4 hours after intercourse C. i should replace my diaphragm every 2 years D. i should use a vaginal lubricant to insert my diaphragm C. i should replace my diaphragm every 2 years The diaphragm is a flexible rubber cup that is filled spermicide and is inserted over the cervix prior to intercourse. The diaphragm is a prescribed device fitted by the provider and should be replaced every 2 years. Should remain in place 6 hrs after intercourse should be cleaned with mild soap and water and dried gently. Contraceptive jelly should be applied prior to placing the device into the vagina A nurse is caring for a newborn who has irregular respirations of 52/min with several periods of apea lasting approximately 5 seconds. The newborn is pink with acrocyanosis. Which of the following actions should the nurse take? A. administer oxygen B. place the newborn in an isolette C. continue to routinely monitor the newborn D. assess the newborn's blood glucose C. continue to routinely monitor the newborn This newborn is exhibiting a normal resp. rate and rhythm. No additional measures are needed at this time. A nurse is caring for a preterm newborn who is receiving oxygen therapy. Which of the following findings should the nurse identify as a potential complication from the oxygen therapy? A. atelectasis B. retinopathy C. interstitial emphysema D. necrotizing enterocolitis B. retinopathy o2 therapy can cause retinopathy of prematurity, especially in preterm newborns. It is a disorder of retinal blood vessel development in the premature newborn. In newborns who develop retinopathy of prematurity, the vessels grow abnormally from the retina into the clear gel that fills the back of the eye. It can reduce vision or result in complete blindness. Terms in this set (281) A nurse is preparing to provide umbilical cord care of a newborn 12 hours after delivery. The nurse notes moderate bleeding from a blood vessel. Which action should be taken? Check the integrity of the cord clamp A nurse is teaching a pt who is postpartum and breastfeeding. Which statement should the nurse include? A reduction in sexual interest could indicate postpartum depression. A nurse is caring for a client who is at 38 weeks and in the active phase of the first stage of labor. Nurse notes 2 late decelerations of the fetal hear rate during the last 5 contractions. What action should nurse take? Assist pt into Lateral position. A nurse is caring for a client at 34 weeks gestation who presents with vaginal bleeding. Which of the following assessment will indicate whether the bleeding is caused by placenta prevue or and abrupt placenta? Uterine tone A nurse is assessing a client who is at 34 weeks and has a cardiac disorder. The nurse should notify the provider about what finding? Pt reports a frequent cough. A pregnant woman asks, "is it ok to have a few beers?" Which response is correct? TOTAL abstinence from alcohol is recommended. Only $1/month The nurse is teaching a client who is postpartum about the RUBELLA vaccine. Nurse should include... You must not become pregnant fro 28 days after this immunization. When teaching a new parent about circumcision care, which instruction should be included? Avoid using diaper wipes on the site during diaper changes. A nurse is caring for a client who is in labor and has fetal heart tracings of variable decelerations. Nursing action is... Reposition pt from side to side. A nurse is assessing a client who is 3 days postpartum. When examining the clients uterus, which of the following techniques should be used? Measure the height of the funds in finger breadths in relation to the umbilicus. A newborn at 30 weeks gestation. Which complications associated with prematurity? Interaventricular hemorrhage. Pt is receiving morphine via a PCA pump following a C-section. What finding would you report to the provider? Urine output of 20 ml/hr. After assessing an 18 hour old newborn, what finding should be reported to the provider? Yellow tinged skin. Pt recently gave birth and is planning to breast feed. Which action should nurse take? Place unwrapped newborn on the mothers bare chest. Caring for a primigravida pt who is at 8 weeks gestation with twins. Pt states she is having ambivalent feelings about the pregnancy. Correct nursing response... These feelings are quite normal at the beginning of pregnancy. Teaching a pt at 13 weeks gestation about the treatment of incomplete cervix with cervical cerclage. Which statement indicates understanding? I should go to the hospital if I think I am in labor. Nurse is performing telephone triage for several pts. Which should the nurse identify as an expected physiological adaptation to pregnancy? Breast tenderness. Pt in latent stage of labor reporting pain of 4/10. Which nursing action to be taken? Lead the pt in relaxation breathing techniques. Postpartum pt has developed hemorrhagic shock. Which finding is expected? Weak and irregular pulse. Pt delivered vaginally 8 hours ago. Pts funds is 2 fingerbreadths about umbilicus and has shifted to the left. There is a large amount of lochia rubra on the perineal pad. What is nursing first action? Assist the pt to the toilet. Pt is in labor and receiving IV oxytocin. Contractions lasting 3 min each. What action should nurse take? Stop the oxytocin infusion. Discharge instructions to the parent of a newborn. Nurse should include which statement? Crib slats should be less that 2.25 inches apart. A nurse is assessing a client at 37 weeks who has a suspected pelvic fracture due to a blunt abdominal trauma. Expected findings... Uterine Contractions Newborn is receiving treatment for jaundice with phototherapy lights. Which intervention should nurse perform? Turn newborn every 2 hr. Planning care for pt who is postpartum and has cardiac disease. For which order should the nurse seek clarification? Monitor the pt weight weekly. Pt is in early labor who has preeclampsia with severe features. Which intervention should nurse perform? Implement seizure precautions. Nurse is assessing a pt 24 hr after delivery and notes the fundus is 2 cm above the umbilicus. Which action should nurse take? Ambulate pt to the bathroom. Nurse is caring for a pt in second stage of labor. Which action should be taken? Promote active movement in and out of bed. Planning care for pt who is 35 weeks. Which lab test should be obtained? Group B strep... Which of the following newborns should the nurse identify as requiring intervention? Newborn who's axillary temp is 36.1C (96.9 F) Nurse reviewing lab results of a pt at 33 weeks. Which result should notify the provider? Platelet 135,000 Immediately after delivery, which action should the nurse perform first for the baby? Place newborn on moms chest. Nurse is teaching about facilities safety measures. Which should be included? Check for photo ID before allowing a nurse to leave the room with the baby. Pt has clinical manifestations of an ectopic pregnancy. What is a risk? PID (pelvis inflammatory disease) Discharge instructions for a pt breastfeeding her baby. Include which statement? Notify provider if notice cracks in nipples. Pt missed 2 cycles and thinks she might be pregnant. Which is positive sign of pregnancy? Presence of HCG in urine. Pt is pregnant and has low progesterone. Which complication is expected. Preterm labor Nurse caring for pt 8 hr postpartum. Which factor places pt at risk of uterine atony? (SATA) Magnesium sulfate infusion Distended bladder Prolonged labor Pt is in labor. Which method will determine the frequency of contractions? Beginning of one contraction to the beginning of the next contraction. Pt reports her episiotomy incision is pulling and stinging. Nursing action... Provide a site bath with warm water. Newborn requires phototherapy for hyperbilirubinemia. Which should nurse include in plan of care? Ensure eyes are closed before applying the eye shield. Nurse administers betamethasone to a client who is 33 weeks to stimulate fetal lung maturity. What is adverse effect of this medication? Decreased blood glucose Which statement should nurse include in discharge teaching for parents of a newborn? Rear facing car seat until 2 years old. Pt is to undergo an amniocentesis to assess fetal lung maturity. The pt is G2P1 and at 36 weeks gestation. She is O+ blood type. Which intervention should be performed? Apply external fetal monitor to pt. Teaching for a postpartum pt who is breastfeeding. Your milk supply with noticeably increase around the 3rd or 4th day after delivery. A nurse is teaching a pt in labor about the use of nitrous oxide for pain control. Which statement indicates understanding? I will feel the effects almost immediately. Nurse teaching pt about Lamaze method to manage pain during labor. Nurse should include what information? You will learn out to prevent pain during labor by focussing your mind to control your breathing. Pt is in active labor and membranes have ruptured. Fetal monitor reveals late decelerations. What is nurses first action? Turn pt onto left side. When assessing a newborn, what finding should be reported to the provider? Central Cyanosis Pt who is postpartum and breastfeeding states "I heard I can't use birth control until I stop breastfeeding". Correct nursing response... A progestin-only pill or injection is available to use. Pt is pregnant and states her last menstrual period (LMP) began on Dec 7. When is patients estimated date of birth (EDB) September 14. (subtract 3 months Then add 7 days) Pt is scheduled to receive a spinal anesthetic. Nurse should plan to perform... Infuse a 500 mL bolus of NS prior to procedure. Teaching for parents on how to care for circumcision at home should include... Encourage non-nutritive sucking for pain relief. Pt who is 28 weeks reports dizziness when lying on her back. Assist pt into what position? Lateral Babies mother received magnesium sulfate to treat preterm labor. Which indicates toxicity in the baby due to this? Respiratory depression Pt has eclampsia and just had a tonic-clonic seizure. After turning head to the side, what should be done next? Give oxygen at 10L via face mask. Baby delivered at 32 weeks. Which finding is anticipated? Extended extremities Pt is at 35 weeks and receiving magnesium sulfate for pre-eclampsia. What should nurse report to the provider? Urinary output of 20 mL/hr What nutrient needs to be increased during actions for breast feeding? Vitamin C Pregnant pt has Type 1 DM. What should include in teaching? You should expect to decrease your insulin dosage immediately after you deliver. Which finding suggests a newborn is post-mature? Nails extending over fingers? Nurse is reviewing lab results during prenatal visit. Hemoglobin is 10g/dL. Which action should be taken? Advise to start Iron and vitamin C supplements. During a pt first prenatal visit, which teaching should nurse include? Doppler can detect HR at 22 weeks. Which newborn is at greatest risk of hypoglycemia? Newborn who is large for gestational age. Pt is 39 weeks. Nurse observes late decelerations in the FHR. What would be the cause? Uteroplacental insufficiency Which would promote optimal developments for a preterm in NICU? Reduce ambient noise and lightning Pt is attempting a trial of labor (TOL) after several C-sections. Pt complains of constant abd pain and prolonged decelerations of the FHR. Action nurse should take? Prepare pt for emergency C-Section. Teaching for proper breast milk storage understanding statement... Discard any unused breastmilk left in the bottle. Newborn has Rx for phototherapy. Mother wonders why. Answer is... The light helps lower babies bilirubin level. What is a complication of an epidural block? Hypotension Pt undergoing a non stress test. FHR is 130/min. with no accelerations for past 10 min. Nursing action... Use vibroacoustic stimulation on the abdomen for 3 sec. Breastfeeding mom has a perineal hematoma. Which breastfeeding position is recommended? Side lying Dad feels resentful of attention mom is getting since pregnant. Nurse response... These feelings are common in early pregnancy. Pt 14 hr postpartum, funds is firm and slightly deviated to right. Nursing action.. Assist her to empty her bladder Pt just had spontaneous rupture of membranes and bradycardia on the FHR with Prolapsed umbilical cord. First action... Manually apply upward pressure intravaginally on the presenting part. Pt had genitalia mutilation 2/2 cultural practice. because of this... Giving birth vaginally my not be possible. Pt is 35 weeks with sever pre-eclampsia. Which provides most accurate information regarding the clients fluid and electrolyte status? Daily weight. Which psychological tasks should the client be expected to accomplish if first trimester? Accepting the pregnancy "How will I know if I am in true or false labor cramps?" True contractions begin irregularly and then become regular in timing. When assessing a pt with placenta previa, which is expected finding? Painless bright red bleeding Newborn is experiencing opioid withdrawal. Which manifestations are expected and should be reported to provider? Tremors Nurse is explaining location suppression. Indicates understanding... I should wear a support bra for a few days. Planning care for pt in labor with HIV. Nursing action after baby is born... Administer the Hep B vaccine prior to discharge. Assessing 2 day old newborn...finds egg-shaped, edematous, bluish discoloration that does not cross the suture line. What info to tell to the mother about this... This will resolve in 3-6 weeks without treatments. Pt is postpartum and having trouble voiding. Nursing action first... Assist pt to bathroom Instructions about self-care for common discomforts during pregnancy... Consume frequent snacks to decrease nausea When assessing newborn for congenital hip dysplasia, finding to expect... Limited abduction of a hip. Which allergy is contraindicated for giving a Hep B vaccine? Bakers Yeast Which Rx should be clarified with Dr. for a pt at 33 weeks with placenta previa and bleeding? Perform vaginal examination. Nurse providing teaching about selection of commercial formula. Should include what? Cow's milk based formula is recommended for healthy newborns. Nurse is preparing to massage the funds of a postpartum with uterine atony. What order should nurse perform fundal massage? Ask client to lie on back with knees flexed. Place hand just above symphysis pubis Position had around top of fundus Rotate upper hand to massage uterus. Use slight downward pressure to compress fundus. During first prenatal visit, pt informs she is lactose intolerant. Which food is good source of calcium? Collard Greens Pt receiving magnesium sulfate by continuous IV infusion. Which medication should be available at the bedside? Calcium Gluconate Pt teaching for promoting optimal development in a premi Provide kangaroo care of the infant. Pt in labor. Vaginal exam reveals: 2 cm, 50% +1, right occiput anterior (ROA). Which fetal position should be documented? Vertex Which finding should be reported to provider in a male newborn? Decreased abdominal movement with breathing. Pt has soft uterus and increased local flow. Which med will promote uterine contractions? Methylergonovine Nurse teaching bout pain management during labor. Statement that pt understands... I can apply heat pack to my back. Nurse is discussing epidural anesthesia with a pt who is receiving oxytocin to induce labor. What statement should the nurse make? An epidural given too early can prolong labor. Nurse reinforcing teaching about preventing engorgement to a pt planning to use formula for newborn includes... Apply ice packs to breasts. Teaching about how to care for circumcision site. I should apply the diaper loosely until the site is healed. Pt is 37 weeks and reports "I became very dizzy while lying in bed this morning, but it went away when I turned on my side". Nursing action... Instruct pt about Vena Cava syndrome and how to prevent it. Pt is 12 weeks and has HIV. What should nurse include in teaching? Continue to take zidovudine during pregnancy. Assessment of a newborn after vaginal birth. Which finding should be reported? Small, pinpoint, reddish purple spots on the chest. Pt in active stage of labor and expresses desire to use non pharmacological pain relief. Which interventions should nurse include? Assist pt into a warm shower. Pt with thrombophlebitis is asking about contraceptive choices. Which should be recommended? Copper uterine device Pt is 12 weeks and practices Hinduism. She needs more protein in her diet. Pt believes eating meat will cause her to miscarry. Nursing response... "Lets discuss other food options that are high in protein you could eat instead of meat." Pt is 12 weeks. Teaching of complications she should report to the provider includes... Facial Swelling. Pt is 32 weeks and experiencing preterm labor. Which medication should nurse administer? Betamethasone Teaching for pt requesting a intrauterine device (IUD). What should nurse include? Risk for ectopic pregnancy increases with IUD. Pt is scheduled to have Prostaglandin E2 gel inserted for cervical ripening. Which action should nurse take? Maintain pt in side-lying position for 30 after insertion. Care actions for pt who delivered a still born baby. Allow parents to keep child in their room as long as they wish. Nutritional teaching for pregnant pt with BMI of 38. Which statement indicates understanding by the pt. I should plan to gain 5-9.1 kg during pregnancy. Which of the following findings on a fetal monitor tracing requires intervention for a pt at 40 weeks. Fetal heart rate of 180/min for 15 min. Teaching for postpartum pt who is breastfeeding. Which statement shows understanding? I should feed baby 8-12 times a day based on feeding cues. When assessing a pt who is receiving magnesium sulfate for pre-eclampsia, which is the priority finding? Urinary output 40 mL in 2 hr. Which of the following structures expels the mature ovum? Graafian follicle Pt is 2 days postpartum. Which location should the nurse locate the fundus? 3 cm below the umbilicus A 30 weeks, warning signs to report to provider include... Vaginal bleeding Pt is 2 hours postpartum and receiving oxytocin IV. The pt asks "why is there so little bleeding?", the correct response is? Bleeding is minimal until I d/c your IV medication. Nurse is administering the Rubella immunization to a pt who is 2 days postpartum. Which statement indicates the pt needs more teaching? I can conceive anytime I want after 10 days. Nurse is assessing the Moro response in a newborn. Which should the nurse expect? Legs move in a similar pattern of response to the arms When a pt is in the 4th stage of labor, the nurse suspects bladder distention. Which finding is expected with this? The bladder fluctuates with palpation. Which should the nurse report when assessing a newborn? Grunting with expiration. When discussing diaphragm use with a pt, which statements indicates understanding of the teaching? I should replace my diaphragm every 2 years. Which intervention should nurse include in the plan of care for a newborn with hyperbilirubinemia? Change baby's position every 2-3 hours Nurse is preparing to administer meperidine hydrochloride to a pt in labor. Which statement should nurse make to pt? This med may make you sleepy. Providing teaching about weight gain to a pt who is a primigravida of normal pregnancy weight. Include which information? Plan to gain 25-35 lbs. Pt is 36 weeks with pre-eclampsia. Which is a priority finding? Nonreactive nonstress test. Pt wants IUD for contraception. Which is contraindicated? Menorrhagia Which lab findings should be reported for a 24 hour old newborn? Hemoglobin 12g/dL (norm= 14-24) Appropriate nursing statement for a couple experiencing infertility issues. You might want to join our support group... Pt is 20 weeks and reports frequent indigestion and heartburn. Instructions... Decrease intake of spicy foods. Which finding should nurse expect for pt at 39 weeks with polyhydramnios? Fetal Gastrointestinal anomaly Which will alert a nurse that a patient is in TRUE labor at 39 weeks? Changes in cervical dilation or effacement. When assessing a newborn circumcised 24 hours ago, which finding should be reported to the provider? The newborn has urinated once since the procedure. (should be 2-6 times) Pt last menstrual period began on July 8. What is the estimated due date? April 15 Which finding should be reported to provider when assessing fetal heart rate tracings while pt is in labor? Late decelerations with fetal bradycardia Which method should be used to obtain a newborns temperature? Axillary Pt in first trimester asks how to manage heartburn. Response... Reduce the amount of food you eat during meals. When assessing a pt at 26 weeks with mild preeclampsia, which finding should be reported to the Dr.? Deep tendon reflexes 4+ Pr has Rx for Naloxone. Which is intended action of med on the CNS? Block effects of narcotics on the CNS Pt is experiencing contractions at 4 min apart. Which pattern should be seen on the fetal monitor tracing? Contractions for 60 sec each with 3 min rest between Which finding indicate a newborn is developing septic shock? Decreased BP Pt is 27 weeks and reports placenta previa and vaginal bleeding. Which other manifestation should be expected? Fundal height measures greater than gestational age. Which should the nurse notify the provider of when assessing a newborn? Mottling Risk factors for developing postpartum depression should include... Gestational diabetes When assessing a newborn, which finding would indicate recent maternal heroin use? Incessant crying Which pt should the nurse identify as a candidate for oral contraceptives? Pt who is breastfeeding a 7 month old infant. (can begin 4 weeks after childbirth) Which reflex is the nurse assessing for by quickly and gently turning the newborns head to one side? Tonic neck. A mother experiencing dystocia looks alarmed and asks, "What's going on? Why are you all poking and prodding? Is my baby okay?" Based on the client's statement, the nurse understands that the client is experiencing which of the following problem? anxiety and fear A nurse is monitoring the status of a client in active labor. The nurse interprets that which finding is consistent with dystocia? Select all that apply. Signs of fetal distress High level of maternal anxiety Failure of the fetus to descend A client arrives at the birthing center in active labor. Her membranes are still intact and the nurse-midwife performs an amniotomy. The nurse explains to the client that after this procedure, she will likely have: increased efficiency of contractions The nurse is assigned to assist with caring for a client who has been admitted to the labor unit. The client is 9 cm dilated and is experiencing precipitous labor. A priority nursing action is to: keep the client in a side lying position A client becomes increasingly more anxious and hyperventilates during the transition phase of labor. The nurse recognizes that the client needs: To regain her breathing pattern A prenatal client with severe abdominal pain is admitted to the labor and birthing department. Which data indicate to the nurse the presence of concealed bleeding? increase in fundal height Only $1/month A nurse is caring for a client in labor. The nurse notes the presence of fetal bradycardia on the fetal monitor and suspects that the umbilical cord is compressed. The nurse immediately places the client in what position? with the lips elevated A nurse is assisting in preparing to care for a client undergoing an induction of labor with an infusion of oxytocin (Pitocin). The nurse includes which of the following in the plan of care? Maintain continuous electronic fetal monitoring. The advantages of using spinal anesthesia for delivery of a fetus include which of the following? Select all that apply. Ease of administration Absence of fetal hypoxia Immediate onset of anesthesia Blockade of sympathetic fibers A client has just had surgery to deliver a nonviable fetus because of abruptio placentae. She has just been told that she is developing disseminated intravascular coagulopathy. She begins to cry and screams, "God, just let me die now!" Which problem should direct care for this client? The client feels hopeless about the situation. A client delivers a viable male neonate who is given APGAR scores of 8 and 9 at 1 and 5 minutes. The nurse determines the physical condition of the neonate to be: good A nurse is caring for a client who was admitted to the maternity unit at 8:00 AM with contractions occurring every 2 minutes, lasting 1½ minutes, and is dilated 4 cm with a cervical effacement of 60%. At 10:30 AM, the contractions cease. The client reports chest pain and manifests signs and symptoms of shock. The nurse quickly plans care, suspecting which of the following? ruptured uterus A nurse assists the nurse-midwife to examine the client. The midwife documents the following data: cervix 80% effaced and 3 cm dilated, vertex presentation minus (−) 2 station, membranes ruptured. The nurse anticipates that the midwife will prescribe which of the following activity for the client? complete bedrest A nurse is caring for a client following a precipitate delivery. In addition to fundal massage, the nurse understands that which nursing action will promote the birth of the placenta? Putting the baby to the mother's breast and letting the baby suck A client tells the nurse her contractions are getting stronger and that she is getting tired. She appears restless, asks the nurse not to leave her alone, and states, "I can't take it anymore." Considering the client's behavior, the nurse suspects she is dilated: 8-10cm A nurse is preparing a client for a cesarean delivery. A urinary catheter is to be inserted into the client's bladder, and the client asks the nurse why this is necessary. The nurse appropriately replies by telling the client that its primary purpose is to: Reduce the risk of injuring the bladder during the surgery. A nurse is caring for a client scheduled for a cesarean delivery. The nurse reviews the client's health record, knowing that which finding needs to be further investigated before delivery? White blood cell count of 35,000 mm3 In providing initial care to the newborn following delivery, the priority action of the nurse is to: Turn the infant's head to the side. A nurse in the delivery room is assisting with the delivery of a newborn. Which observation would indicate that the placenta has separated from the uterine wall and is ready for delivery? Changes in the shape of the uterus At 5:00 AM a client is admitted to the maternity unit after experiencing 3 hours of labor at home. The assessment determines that the fetal heart rate (FHR) is 140 beats per minute with the fetus at station 0 and strong contractions occurring every 3 minutes. It is now 7:00 AM with little progress, and the FHR is decreasing. It is most appropriate for the nurse to anticipate the need to: Prepare the client for a cesarean delivery. A nurse observes that a client in the transition stage of labor is crying out in pain with pushing efforts. The nurse recognizes this behavior as: Fear of losing control The nurse prepares to administer erythromycin ophthalmic ointment to a newborn infant immediately after delivery. The nurse understands that this ointment: Is effective in protecting the newborn from Neisseria gonorrhoeae and chlamydia A nurse is assisting in caring for a client in labor. The nurse recognizes that the risks for uterine rupture during labor and delivery include: shoulder dystocia A nurse in the labor room is caring for a client in the first stage of labor. On assessing the fetal patterns, the nurse notes an early deceleration of the fetal heart rate (FHR) on the monitor strip. Which is the appropriate nursing action? Document the findings and continue to monitor the fetal patterns. A nurse is preparing a client for an emergency cesarean delivery. Which of the following information regarding the client has priority? When was the last time the client ate or drank? The client is having moderate contractions that are occurring every 5 minutes and lasting 60 seconds. The fetal heart rate (FHR) is 150 beats per minute and regular. Based on these findings, what is the appropriate nursing action? Continue to monitor the client. The nurse institutes measures for the client with placental abruption to minimize alterations in fetal tissue perfusion. The nurse determines that fetal tissue perfusion is adequate if which of the following is noted? Presence of accelerations A nurse assisting in the labor room is preparing to care for a client with hypertonic dysfunction. The nurse is told that the client is experiencing uncoordinated contractions that are erratic in their frequency, duration, and intensity. The priority nursing intervention in caring for the client is to: Provide pain relief measures. A nurse notes that a client in labor has foul-smelling amniotic fluid, a maternal temperature of 101° F, and a urine output of 150 mL during the past 2 hours. The nurse should do which of the following at this time? Notify the registered nurse of a possible maternal infection. A pregnant client is receiving magnesium sulfate for the management of preeclampsia. A nurse determines that the client is experiencing toxicity from the medication if which of the following is noted on data collection? Respirations of 10 breaths per minute Which documentation concerning the characteristics of amniotic fluid supports the determination that the fluid is normal? It is pale, straw-colored with flecks of vernix. A nurse is caring for a client in preterm labor when her membranes rupture. The initial nursing action is to: Monitor the fetal heart rate. A nurse is caring for a woman in labor who is experiencing a precipitate delivery. Until help arrives, the nurse places the client into which optimal position? lateral sims' A client in labor asks the nurse why it is so important to void frequently during labor. The nurse responds, using knowledge that the important reason is to: Ensure labor progress and prevent injury. A nurse assisting in the care of a woman in labor should focus primarily on which of the following at the time of delivery? infant A nurse is monitoring a preterm labor client who is receiving magnesium sulfate intravenously. The nurse monitors for which adverse effect(s) of this medication? Select all that apply. Depressed respirations Extreme muscle weakness Flushing A nurse in the labor room is assisting in caring for a client in the active stage of labor. The nurse is told that the fetal patterns show a late deceleration on the monitor strip. Based on this finding, the nurse prepares for which appropriate nursing action? Administering oxygen via face mask A nurse is collecting initial data on a newborn in the delivery room. Which observation would the nurse expect to note when examining the umbilical cord of the newborn? Two arteries and one vein A nurse is assigned to care for a client experiencing dystocia. In planning care, the nurse would consider the highest priority to be frequent: Monitoring for changes in the physical and emotional condition of the mother and fetus A primigravida's membranes rupture spontaneously. The nurse's first action is to: Determine the fetal heart rate. A nurse is providing emergency measures to a pregnant client with a prolapsed cord. The mother becomes anxious and frightened and says to the nurse, "Why are all of these people in here? Is my baby going to be all right?" Which of the following most appropriately describes the mother's problem at this time? Fear about what is happening A client has just delivered a viable newborn. The first nursing action to initiate attachment is to: Determine the parents' desires for contact with the newborn. A client asks, "What does it mean that the baby is at minus one?" The nurse should explain to the client that the fetal presenting part is isolated: 1 cm above the ischial spines A licensed practical nurse (LPN) is assisting in gathering data on a client who is scheduled for a cesarean delivery. Which of the following findings would indicate a need to contact the registered nurse (RN)? Fetal heart rate of 180 beats per minute A nurse assists in developing a plan of care for a multigravida client who has a history of cesarean birth. It is determined that the client is at high risk of uterine rupture. The nurse plans to monitor the client closely for: signs of shock A client is scheduled to have an elective cesarean delivery. The nurse preparing the client for the procedure plans to allay the client's feelings of anxiety by: Encouraging the client to discuss her concerns and desires regarding anesthesia options A nurse assisting to monitor a client in labor is told that the client's cervix is 3 cm dilated with contractions occurring every 2 to 3 minutes. When monitoring the client's psychological status, the nurse anticipates the client to reflect an attitude of: excitement A nurse is reviewing the record of a client in the labor room and notes that the nurse midwife has documented that the fetus is at minus one station. The nurse determines that the fetal presenting part is: 1 cm above the ischial spines A nurse caring for a client who is receiving oxytocin (Pitocin) for the induction of labor notes a nonreassuring fetal heart rate (FHR) pattern on the fetal monitor. On the basis of this finding, the nurse would first: Stop the oxytocin infusion. The client is in the second stage of labor. As the baby begins to crown, the health care provider administers a pudendal nerve block in preparation for an episiotomy. The nurse should: Continue to assess vital signs and fetal heart rate the same as before the nerve block. A multigravida woman with a history of cesarean births is admitted to the maternity unit in labor. The client is having excessively strong contractions, and the nurse monitors the client closely for uterine rupture. Which finding would be noted if complete rupture occurs? Decreasing blood pressure The nurse is assigned to assist with caring for a client who is being admitted to the birthing center in early labor. On admission, the nurse would initially: Determine the maternal and fetal vital signs. A nurse is reviewing the record of a client in the labor room and notes that the nurse-midwife has documented that the fetus is at minus one station. The nurse determines that the fetal presenting part is: 1 cm above the ischial spines A client in preterm labor is placed on bedrest. The nurse assists the client to which of the following advantageous positions? left lateral A nurse is assisting in performing Leopold's maneuvers. When the client asks what these are for, the nurse's best response is that these maneuvers help to determine: Fetal position A nurse tells a client she is now beginning the second stage of labor. The nurse realizes the client understands the occurrences of this stage when the client says: "My cervix is completely dilated." A nurse is monitoring a client who is in the active stage of labor. The nurse notes a late deceleration on the fetal monitor. Based on this observation, the nurse immediately: Administers oxygen via face mask to the mother A nurse prepares to explain the purpose of effleurage to a client in early labor. The nurse tells the client that effleurage: Is light stroking of the abdomen to facilitate relaxation during labor A client in labor is transported to the delivery room and is prepared for a cesarean delivery. The client is positioned on the delivery room table and the nurse places the client in the: Supine position with a wedge under the right hip A nurse is evaluating the effectiveness of meperidine hydrochloride (Demerol) for pain management for a client in labor. The client describes her pain level as "9" during contractions. The nurse determines that the medication was effective if the client exhibited which reasonable goal for pain relief? Pain level is "4" while a progressive labor pattern continues. A nurse reviews the results of an ultrasound performed on a woman admitted to the maternity unit. The results indicate that the placenta is covering the entire internal cervical os. The nurse understands that the client is experiencing: Complete placenta previa Following delivery, a client experiences subinvolution of the uterus. The nurse develops a plan of care, recalling that which of the following is the primary cause for this occurrence? Retained placental fragments A client in active labor with intact membranes is complaining of back discomfort. An analgesic was administered 1 hour ago but has not relieved the discomfort. The nurse should avoid doing which of the following at this time to assist in relieving the back discomfort? Assist the client to ambulate in the room. A nurse is monitoring a client who is receiving oxytocin (Pitocin) to augment labor. The nurse determines that the dosage should be decreased and notifies the registered nurse if which of the following is noted? fetal tachycardia The maternity nurse prepares the client for which of the following techniques commonly used to relieve shoulder dystocia? McRoberts' maneuver Immediately following the delivery of a newborn, the nurse prepares to assist in the delivery of the placenta. What is the appropriate action to deliver the placenta? Pull gently on the cord as the mother bears down. A client in labor has been pushing effectively for 1 hour and the presenting part is at a +2 station. The nurse determines that the client's primary physiological need at this time is: Rest between contractions A nurse is caring for a client with a diagnosis of dystocia. The nurse specifically collects data regarding which of the following? Characteristics of contractions A nurse collecting data on a client during the second stage of labor notes a slowing of the fetal heart rate (FHR) with a loss of variability and determines that these are indicators of possible complications. Which priority interventions should the nurse perform? Turn client to her side and administer oxygen by mask at 8 to 10 L/min A 30-week gestational prenatal client with complaints of painless vaginal bleeding presents at the labor and birthing department of the hospital. The nurse prepares the client for which expected diagnostic procedure? Contraction stress test A nurse is monitoring a client in active labor and notes that the client is having contractions every 3 minutes that last 45 seconds. The nurse notes that the fetal heart rate between contractions is 100 beats per minute. Which of the following nursing actions is appropriate? Notify the registered nurse (RN). A woman in active labor has contractions every 2 to 3 minutes that last for 45 seconds. The fetal heart rate between contractions is 100 beats per minute. On the basis of these findings, the priority nursing intervention is to: Notify the registered nurse (RN) immediately. During the intrapartum period, a nurse is caring for a laboring client with sickle cell disease. The nurse ensures that the client receives appropriate intravenous (IV) fluid intake and oxygen consumption to primarily: Prevent dehydration and hypoxemia. When examining the umbilical cord immediately after birth, the nurse expects to observe: two arteries Before attempting to deliver the placenta after a precipitate delivery, the nurse waits for which sign as an indication of placental separation? Change in uterine shape A nurse is caring for the nullipara woman in labor. The nurse understands that the health care provider must be contacted if which one of the following becomes apparent? Decreased periods of uterine relaxation between contractions The client is admitted to the labor suite complaining of painless vaginal bleeding. The nurse assists with the examination of the client, knowing that a routine labor procedure that is contraindicated with this client's situation is: A manual pelvic examination Leopold's maneuvers will be performed on a pregnant client. The client asks the nurse about the procedure. The nurse responds, knowing that this procedure: Is a systematic method for palpating the fetus through the maternal abdominal wall A client who experienced abruptio placentae is at risk for disseminated intravascular coagulopathy (DIC). The nurse should monitor this client for which manifestation of this complication? Oozing from injection sites The nurse is collecting data from a client who has been diagnosed with placenta previa. Choose the findings that the nurse would expect to note. Select all that apply. Bright red vaginal bleeding Soft, relaxed, nontender uterus A nurse is caring for a client in labor. The fetal heart rate is 156 beats per minute and regular. The client's contractions are occurring every 4 minutes with a duration of 42 seconds and moderate intensity. The nurse should do which of the following at this time? Continue monitoring the client because the data reflect acceptable progress. A client is admitted for an emergency cesarean section delivery. Contractions are occurring every 15 minutes. The client has a temperature of 100° F and ate 2 hours ago. Which intervention has priority? Report the time of last food intake to the health care provider. A nurse has assisted in developing a plan of care for a client experiencing dystocia and includes several nursing interventions in the plan of care. The nurse prioritizes the plan and selects which nursing intervention as the highest priority? Monitoring fetal status The nurse is caring for a client who is in labor. The nurse rechecks the client's blood pressure and notes that it has dropped. To decrease the incidence of supine hypotension, the nurse should encourage the client to remain in which position? Left lateral A pregnant client at 36 weeks' gestation experiences painless bleeding and is admitted to the labor room. Which action should the nurse initially include in the plan of care? Maintain complete bedrest, monitor IV fluid intake, and monitor the fetal heart rate. A pregnant client with severe uterine bleeding is admitted to the labor and birthing department. Which of the following data would best alert the nurse to early signs of hypovolemic shock? Restlessness and agitation A 31-week preterm labor client dilated to 4 centimeters has been started on magnesium sulfate. Her contractions have stopped. If the client's labor can be inhibited for the next 48 hours, what medication does the nurse anticipate will be prescribed? Betamethasone The client at 38 weeks' gestation is admitted to the birthing center in early labor. The client is carrying twins, and one of the fetuses is in a breech presentation. The nurse assists with planning care for the client and identifies which of the following as the lowest priority for the care of this client? Measuring the fundal height The nurse is assigned to assist with caring for a client with abruptio placentae who is experiencing vaginal bleeding. The nurse collects data from the client, knowing that abruptio placenta is accompanied by which additional finding? Uterine tenderness on palpation A client in labor has an underlying diagnosis of sickle cell anemia. During labor the client is at high risk for sickling crisis. The nurse should take which priority action to assist in preventing a crisis from occurring during labor? Administer oxygen as prescribed throughout labor. A nurse is monitoring a client in labor whose membranes rupture spontaneously. The initial nursing action is to: Determine the fetal heart rate. A woman with preeclampsia is receiving magnesium sulfate. The nurse assigned to care for the client determines that the magnesium sulfate therapy is effective if: Seizures do not occur. For the previous 4 hours, a client in labor has been experiencing contractions every 2 minutes, lasting 60 to 70 seconds, and strong to palpation. She is 2 cm dilated and complaining of severe pain. The nurse understands that the client is experiencing which type of dystocia? Hypertonic The nurse is assigned to care for a client who is in early labor. When collecting data from the client, it is most important for the nurse to first determine which of the following? Baseline fetal heart rate A client was admitted to the maternity unit 12 hours ago and has been experiencing strong contractions every 3 minutes, and the fetus is currently at station 0. The fetal heart rate on admission was 140 beats per minute and regular. The fetal heart rate is decreasing and a persistent nonreassuring fetal heart rate pattern is present. What is the appropriate nursing action? Prepare the client for a cesarean delivery. A nurse is asked to assist the primary health care provider in performing Leopold's maneuvers on a client. Which nursing intervention should be implemented before this procedure is performed? Have the client empty her bladder. A client has been admitted to the maternity unit for a scheduled cesarean section. As she is getting into bed for preliminary preparation for surgery, the client states, "I don't need the cesarean section after all because I think my baby has moved around." The appropriate response by the nurse is which of the following? "Tell me what you mean when you say that your baby has moved." A nurse is assisting in caring for a client with abruptio placentae and is monitoring the client for disseminated intravascular coagulopathy (DIC). Which of the following findings is least likely associated with DIC? Swelling of the calf of one leg The client who is being prepared for a cesarean delivery is brought to the delivery room. To maintain the optimal perfusion of oxygenated blood to the fetus, the nurse places the client in the: Supine position with a wedge under the right hip A nurse is caring for a client with sickle cell disease who is in labor. The nurse ensures that the client receives appropriate intravenous (IV) fluid intake and oxygen consumption to primarily: Prevent dehydration and hypoxemia. [Show More]

Last updated: 1 year ago

Preview 1 out of 76 pages

Reviews( 0 )

$17.00

Add to cart

Instant download

Can't find what you want? Try our AI powered Search

OR

GET ASSIGNMENT HELP
118
0

Document information


Connected school, study & course


About the document


Uploaded On

Jun 09, 2021

Number of pages

76

Written in

Seller


seller-icon
A+ Solutions

Member since 3 years

164 Documents Sold


Additional information

This document has been written for:

Uploaded

Jun 09, 2021

Downloads

 0

Views

 118

Document Keyword Tags

Recommended For You


$17.00
What is Browsegrades

In Browsegrades, a student can earn by offering help to other student. Students can help other students with materials by upploading their notes and earn money.

We are here to help

We're available through e-mail, Twitter, Facebook, and live chat.
 FAQ
 Questions? Leave a message!

Follow us on
 Twitter

Copyright © Browsegrades · High quality services·